Chapter 5. Diagnosis of Pain States Flashcards

1
Q
  1. A59-year-old female comes to your office complaining
    of moderately severe low back pain
    and right buttock pain which is exacerbated
    with prolonged sitting. On physical examination
    there is sciatic notch tenderness and the
    pain is exacerbated with flexion, adduction,
    and internal rotation of the right hip. Which of
    the following is the most likely diagnosis?
    (A) L5-S1 facet syndrome
    (B) Piriformis syndrome
    (C) Sacroiliac (SI) joint syndrome
    (D) Sciatica
    (E) L3 radiculopathy
A
  1. (B) The piriformis syndrome was originally
    described by six common characteristics
    (1) trauma; (2) pain in the muscle with sciatica
    and difficulty in walking; (3) worsening with
    squatting or lifting; (4) a sausage-like mass
    within the muscle; (5) positive Lasègue sign;
    and (6) gluteal atrophy. The female to male
    ratio is 6 to 1.
    There are many approaches to evaluate
    piriformis syndrome. One method is in the sitting
    position which involves the examiner
    stretching the piriformis muscle by passively
    moving the hip into internal rotation reproducing
    buttock pain which is relieved by the
    examiner passively moving the hip into external
    rotation. The patient then actively rotates
    the hip against the resistance which reproduces
    buttock pain. Furthermore, there is generally
    point tenderness on palpation of the
    belly of the piriformis muscle. There tends to
    be prolongation of the H-reflex with flexion,
    adduction, and internal rotation.
How well did you know this?
1
Not at all
2
3
4
5
Perfectly
2
Q
  1. A77-year-old female comes to your office complaining
    of 6 months of severe right buttock
    pain radiating into the right lower leg. The pain
    is also present at night and not uncommonly
    interferes with sleep. The pain is severe with
    sitting or lying on her back or right side, however,
    quickly dissipates with normal erect posture.
    Which of the following is the most likely
    diagnosis?
    (A) Snapping bottom
    (B) Sciatica
    (C) Radiculopathy
    (D) Piriformis syndrome
    (E) Weaver’s bottom
A
  1. (E) In classic weaver’s bottom (ischiogluteal
    bursitis)—the patients invariably get pain sitting
    which goes away upon standing or lying
    on their contralateral side. However, the pain
    promptly returns upon resuming a seated position.
    Typically, the patient can consistently
    point to the spot where it hurts with their finger
    and state “it hurts right here.” On physical
    examination, tenderness is evoked with palpation
    over the ischiogluteal bursa.
How well did you know this?
1
Not at all
2
3
4
5
Perfectly
3
Q
  1. A 53-year-old male comes to your office complaining
    of foot pain (predominantly in the
    heel—but also with diffuse plantar symptoms)
    which also occurs at night and can be exacerbated
    by prolonged standing or walking. It is
    associated with weakness of the phalanges
    (impairing the pushing off phase of walking) as
    well as sensory loss and paresthesia. After a
    complete history and physical examination are
    completed, which of the following is the next
    most appropriate step?
    (A) Magnetic resonance imaging (MRI) of
    the ankle
    (B) MRI of the lumbar spine
    (C) Initiate anti-inflammatory medications
    (D) Trial of arch support
    (E) Electrodiagnostic testing
A
  1. (E) Imaging studies are most appropriate with
    bony point tenderness or when the differential
    diagnosis is likely calcaneal stress, fracture
    Paget disease, tumors, calcaneal apophysitis
    (Sever disease in adolescents), or calcaneal
    stress fracture. The most appropriate diagnostic
    evaluation for suspected tarsal tunnel syndrome
    is electrodiagnostic evaluation.
How well did you know this?
1
Not at all
2
3
4
5
Perfectly
4
Q
  1. A 53-year-old male comes to your office complaining
    of foot pain (predominantly in the
    heel—but also with diffuse plantar symptoms)
    which also occurs at night and can be exacerbated
    by prolonged standing or walking. It is
    associated with weakness of the phalanges
    (impairing the pushing off phase of walking) as
    well as sensory loss and paresthesia. Which of
    the following is the most likely diagnosis?
    (A) Morton neuroma
    (B) Peripheral neuropathies
    (C) Medial plantar nerve entrapment
    (D) Tarsal tunnel syndrome
    (E) March fracture
A
  1. (D) The tarsal tunnel located behind and inferior
    to the medial malleolus. It is bounded on
    the lateral aspect by the tibia and medially by
    the flexor retinaculum (laciniate ligament). Its
    contents include the tibial nerve, posterior tibial
    tendons, flexor digitorum longus tendon, flexor
    hallucis longus tendon, tibial artery, and tibial
    vein. Within the tarsal tunnel or immediately
    distal to it, the tibial nerve divides into the
    medial and lateral plantar nerves. The calcaneal
    branch originates variably above or below the
    flexor retinaculum to supply the heel and calcaneal
    skin. The tarsal tunnel syndrome most
    commonly arises from trauma (eg, fractures,
    ankle dislocations) and is characterized by foot
    pain and paresthesia, as well as potentially by
    sensory loss and Tinel sign at the ankle. The
    pain may be similar to carpal tunnel syndrome
    in that it often occurs at night. Furthermore, it
    may be exacerbated by prolonged standing or
    walking. Amarch fracture is a stress fracture of
    the metatarsal bone. The second and third
    metatarsals are the most common sites. Patients
    complain of increased intensity of pain with
    activity or exercise. The pain is localized to the
    site of the fracture.
How well did you know this?
1
Not at all
2
3
4
5
Perfectly
5
Q
  1. A47-year-old female comes to your office complaining
    of an aching forearm with discomfort
    and numbness in the thumb and index finger,
    and weakness in the hand. Apositive Tinel sign
    is present in the forearm. Which of the following
    is the most likely diagnosis?
    (A) Anterior interosseous nerve syndrome
    (B) Posterior interosseous nerve syndrome
    (C) Ulnar nerve entrapment
    (D) Pronator syndrome
    (E) Radial nerve entrapment
A
  1. (D) Pronator syndrome may result from compression
    of the median nerve proximal to the
    branching of the anterior interosseous nerve.
    Patients with pronator syndrome generally
    complain of an aching discomfort of the forearm,
    numbness in the thumb and index finger, and weakness in the hand. On physical examination
    there may be tenderness over the proximal
    part of the pronator teres muscle that is
    exacerbated by pronation of the forearm
    against resistance. Resisted pronation may also
    result in paresthesia in the distribution of the
    median nerve. A positive Tinel sign is often
    present at the proximal edge of the pronator
    muscle. If the entrapment is under the bicipital
    aponeurosis this may result in weakness of the
    pronator muscle and depending on the degree
    of compression, weakness of other muscles (eg,
    long flexor muscles of the fingers and thumb,
    abductor pollicis brevis)
How well did you know this?
1
Not at all
2
3
4
5
Perfectly
6
Q
257. Complex regional pain syndrome type II
(CRPS II) differs from CRPS I because in CRPS II
there is
(A) allodynia
(B) movement disorder
(C) sudomotor and vasomotor changes
(D) evidence of major nerve damage
(E) severe swelling
A
  1. (D) CRPS I and CRPS II are clinically indistinguishable.
    The only difference is that in CRPS II
    there is evidence of major nerve damage.
How well did you know this?
1
Not at all
2
3
4
5
Perfectly
7
Q
258. Which of the following range is the temperature
most appropriate to use as a stimulus
when evaluating warm temperature sensation?
(A) 25°C to 30°C
(B) 30°C to 35°C
(C) 35°C to 40°C
(D) 40°C to 45°C
(E) 45°C to 50°C
A
  1. (D) The temperature range to test warm temperature
    sensation is 40°C to 45°C—usually
    done via a glass or metal tube with hot (40°C-
    45°C) water. Temperatures higher than 45°C
    are generally perceived as painful.
How well did you know this?
1
Not at all
2
3
4
5
Perfectly
8
Q
259. Which of the following range is the temperature
most appropriate to use as a stimulus
when evaluating cold temperature sensation?
(A) −5°C to 0°C
(B) 0°C to 5°C
(C) 5°C to 10°C
(D) 10°C to 15°C
(E) 15°C to 20°C
A
  1. (C) The temperature range to test cold temperature
    sensation is 5°C to 10°C—which may be
    done with a thermophore. Temperatures lower
    than 5°C are generally perceived as painful.
How well did you know this?
1
Not at all
2
3
4
5
Perfectly
9
Q
260. Which of the following may potentially facilitate
or perpetuate myofascial trigger points in
some patients?
(A) Low creatine kinase
(B) Low aldolase
(C) Low cholesterol
(D) Low vitamin D
(E) Low vitamin B12 or folate
A
  1. (E) Low levels of vitamin B12 and/or folate may
    be associated with increased trigger points in
    many patients who suffer from myofascial pain
    syndrome. Multiple coexisting systemic conditions
    may also be associated with myofascial
    pain syndrome and should be investigated
    in patients with severe painful myofascial trigger
    points.
How well did you know this?
1
Not at all
2
3
4
5
Perfectly
10
Q
  1. A 39-year-old male with persistent coughing
    attributed to upper respiratory infection (URI)
    comes to your office complaining of moderate
    anterior chest wall pain—it is only on the left
    side—predominantly over the second and third
    costal cartilages. Bulbous swellings and point
    tenderness are present at these sites. Which of
    the following is the most appropriate diagnosis
    for this patient?
    (A) Intercostal neuralgia
    (B) Tietze syndrome
    (C) Acute myocardial infarction
    (D) Pneumonia
    (E) Pleurisy
A
  1. (B) Tietze syndrome (costochondritis) should
    only be diagnosed after other diagnoses are
    ruled out. It is most frequently unilateral
    involving the second and third costal cartilages
    and is characterized by mild to moderately
    severe anterior chest wall pain. The pain is typically
    localized in the region of the costal cartilages
    but may occasionally radiate to the arm
    and shoulder. Tietze syndrome occurs more
    commonly under the age of 40 years. On
    physical examination, tenderness to palpation
    as well as bulbous swelling over the costochondral
    junctions may be found.
How well did you know this?
1
Not at all
2
3
4
5
Perfectly
11
Q
  1. A 66-year-old woman who did not have a history
    of trauma comes to your office complaining
    of acute, severe, constant medial right knee
    pain for 6 weeks. MRI imaging demonstrated
    extensive narrow edema of the medial femoral
    condyle with significant soft tissue edema
    around the superficial and deep compartment
    of the medial collateral ligament (MCL) but
    without MCL disruption. Which of the following
    is the most likely diagnosis?
    (A) Stress fracture
    (B) MCL tear
    (C) Medial meniscal tear
    (D) Spontaneous osteonecrosis of the
    knee (SONK)
    (E) Medical femoral condyle contusion
A
  1. (D) Spontaneous osteonecrosis of the knee
    (SONK) is an entity whose precise pathogenesis
    remains unclear. The pain may be present at
    rest and is generally well-localized without
    trauma or associated incited event. It is classically
    defined as unilateral and spontaneous
    with predilection for the medial femoral
    condyle. It occurs typically in the elderly population
    (> age 60) and is three times more
    common in women than men. Initial radiographs
    tend to be normal.
How well did you know this?
1
Not at all
2
3
4
5
Perfectly
12
Q
  1. A 49-year-old male comes to your office after
    climbing several mountain passes in the
    Pyrenees on a bicycle with thigh complaints.
    He relates to you that he developed a painful
    sensation on the lateral aspect of his right thigh,
    which lasted for about a week. This was followed
    by numbness and paresthesia in the
    same location. Physical examination revealed
    sensory loss in the same location. Which of the
    following is the most likely diagnosis?
    (A) Tensor fascia lata syndrome
    (B) Meralgia paresthetica
    (C) Iliotibial band syndrome
    (D) Greater trochanteric bursitis
    (E) Lumbar radiculopathy
A
  1. (B) Meralgia paresthetica is a painful mononeuropathy
    of the lateral femoral cutaneous nerve
    (LFCN). Although it may be idiopathic in
    nature it is commonly caused by focal entrapment
    of the LFCN as it passes through the
    inguinal ligament. Although there have been
    numerous reported associated conditions,
    some of these include weight change (eg, obesity,
    pregnancy), possibly external compression
    (eg, seat belts, tight clothing), perioperative
    factors/trauma, retroperitoneal tumors, and
    strenuous walking/cycling (the iliopsoas muscle
    and tensor fascia lata are heavily involved in
    walking and/or cycling movement).
How well did you know this?
1
Not at all
2
3
4
5
Perfectly
13
Q
  1. A43-year-old male runner comes to your office
    complaining of a dull ache in the anterior
    aspect of the lower legs bilaterally which occurs
    about 10 minutes into his running routine each
    time he runs and dissipates with rest. The
    patient states that he needs to stop running
    because of this ache and also notes dysesthesia
    in the first web space of both feet. Which of
    the following is the most likely diagnosis?
    (A) Shin splints
    (B) Stress fractures
    (C) Chronic osteomyelitis
    (D) Periostitis
    (E) Chronic exertional anterior compartment
    syndrome of the lower leg
A
  1. (E) Chronic exertional compartment syndrome
    of the anterior tibial compartment may occur in
    runners, soccer players, and racers and may
    present with a fullness in the anterior compartment,
    exacerbation of pain on passive dorsiflexion
    of the great toe, weakness of the
    extensor hallucis longus muscle, and decreased
    sensation in the first web space. Symptoms are
    usually bilateral 75% to 95% of the time
How well did you know this?
1
Not at all
2
3
4
5
Perfectly
14
Q
  1. A32-year-old construction worker felt a sharp
    pain in his back radiating down to the heel of
    his right foot after lifting a large, metal girder.
    Two days later he noticed numbness in the sole
    of his right foot and fifth toe. Physical examination
    is notable for a decreased ability to walk
    on his toes, a positive straight leg raising test on
    the right, and a markedly diminished ankle
    jerk reflex. Which of the following is the most
    likely diagnosis?
    (A) L4-5 herniated disc
    (B) Discogenic low back pain
    (C) L5-S1 herniated disc
    (D) Spinal stenosis
    (E) Piriformis syndrome
A
  1. (C) Symptoms from an L5-S1 herniated disc
    are typically experienced in the distribution of
    the S1 nerve root. These symptoms may
    include pain or sensory changes in the calf, lateral
    border of the foot, heel, sole, and sometimes
    fourth and fifth toes. On physical
    examination, the patient may have diminished
    strength in the gastrocnemius, soleus, and the
    peroneus longus and brevis muscles. An L4-5
    herniated disc most frequently results in L5 symptoms, which include diminished sensation
    in the lateral leg, dorsum of the foot, and
    the first two toes. Spinal stenosis is narrowing
    of the spinal canal that occurs with aging.
    Patients may present with decreased strength
    and loss of sensation, but with central stenosis
    it is usually nondermatomal. Piriformis syndrome
    is an uncommon cause of buttock pain
    and/or sciatica that is caused by sciatic nerve
    compression by the piriformis muscle. Although
    sciatica is often present, pain from piriformis
    syndrome is nonadicular, and hence straight
    leg raising tests should not be positive.
    Discogenic pain is pain that results from internal
    disc disruption. The neurologic examination
    should be nonfocal when pain results
    solely from internal disc derangement.
How well did you know this?
1
Not at all
2
3
4
5
Perfectly
15
Q
  1. An 80-year-old man presents with a 2-year
    history of low back pain radiating down from
    both legs to his ankles. He also notes numbness
    in his left foot and slight weakness. The pain is
    increased with walking and relieved within
    seconds of cessation of activity. Leaning forward
    eases his pain and lying supine relieves it.
    Which of the following is the most likely
    diagnosis?
    (A) Herniated nucleus pulposus
    (B) Facet arthropathy
    (C) Muscle spasm
    (D) Arachnoiditis
    (E) Spinal stenosis
A
  1. (E) As we age, our spinal canal starts to narrow.
    This narrowing is a result of many different
    processes including disc bulging from a progressive
    loss of disc height and elasticity,
    hypertrophy of the facet joints and ligamentum
    flavum and osteophyte formation.
    Technically, the term “spinal stenosis” can refer
    to central canal stenosis, lateral recess stenosis,
    or foraminal stenosis. The typical presentation
    of someone with spinal stenosis is an elderly
    person with low back and leg pain brought on
    by walking, especially on stairs or hills.
    Frequently, the pain is bilateral. In contrast to
    vascular claudication, patients with neurogenic
    or pseudoclaudication often find that the cessation
    of walking brings immediate pain relief.
    Like spinal stenosis, facet arthropathy is more
    common in the elderly, but the pain does not
    typically radiate into the lower leg and is usually
    not associated with loss of sensation.
How well did you know this?
1
Not at all
2
3
4
5
Perfectly
16
Q
  1. A 31-year-old woman presents to your office
    with marked pain and swelling in her ankle
    6 weeks after an open reduction internal fixation
    with casting. On examination, the ankle is
    warm and erythematous. Lightly touching the
    ankle with a cotton swab evokes severe, lancinating
    pain. You suspect CRPS I. Which of the
    following tests will confirm your diagnosis?
    (A) Lumbar sympathetic block
    (B) Phentolamine infusion test
    (C) Triple phase isotope bone scan
    (D) Erythrocyte sedimentation rate
    (E) None of the above
A
  1. (E) In the early 1990s, a panel of experts
    reached a consensus that the terms “reflex sympathetic
    dystrophy” and “causalgia” had lost
    their utility as clinical diagnoses and suggested
    a new nomenclature be adopted. The new
    terms designated for these conditions are “CRPS
    types I and II”. According to the new diagnostic
    criteria, CRPS need not be maintained by
    sympathetic mechanisms. A three-phase isotope
    bone scan is often positive in CRPS, but a
    normal bone scan does not exclude the diagnosis.
    Erythrocyte sedimentation rate is a nonspecific
    test that is positive in many painful conditions
    including infection, inflammatory arthritides
    and inflammatory myopathies. As a syndrome,
    CRPS is diagnosed by history and physical
    examination. For CRPS I, the diagnostic criteria
    include (1) an initiating noxious event; (2) spontaneous
    pain and/or allodynia occur outside
    the territory of a single peripheral nerve, and
    are disproportionate to the inciting event; (3)
    there is or has been evidence of edema, cutaneous
    perfusion abnormalities, or abnormal
    sudomotor activity, in the region of pain since
    the inciting event; and (4) the diagnosis is
    excluded by the existence of any condition that
    would otherwise account for the degree of pain
    and dysfunction.
How well did you know this?
1
Not at all
2
3
4
5
Perfectly
17
Q
  1. A 46-year-old man complains of worsening
    back and new onset leg pain and paresthesia
    10 weeks after an L4-S1 posterior spinal fusion.
    One week after the surgery, the patient
    reported 85% pain relief. Which radiologic test
    would be most appropriate for detecting the
    cause of failed back surgery syndrome (FBSS)
    in this patient?
    (A) Computed tomographic (CT) scan with
    contrast
    (B) Myelography
    (C) Epidural mapping via the injection of
    contrast under fluoroscopy through a
    catheter inserted through the caudal
    canal
    (D) T2-weighted MRI with contrast
    (E) Further radiologic study is not indicated
    at this point
A
  1. (D) The type and timing of pain after spine surgery
    provide important clues as to the possible
    diagnosis. For example, no change in a patient’s
    pain pattern after surgery may indicate that
    either the wrong surgery was done or the procedure
    was technically unsuccessful. In this
    case, the patient experienced initial pain relief,
    which was followed by worsening back pain
    and new-onset leg pain several weeks later.
    Possible causes of this scenario include epidural
    fibrosis, arachnoiditis, discitis, battered root
    syndrome with perineural scarring, or an early
    recurrent disc herniation. Pseudoarthrosis,
    juxtafusional discogenic pain, and lumbar instability
    can also be causes of FBSS, but in these
    cases the recurrence of pain typically occurs
    much later. For detecting disc pathology, MRI is
    more sensitive than CT or myelography. It is
    also more sensitive than CT for identifying contrast
    enhancement. For the possible etiologies
    that fit this patient’s pain history (ie, arachnoiditis,
    epidural fibrosis, and discitis), contrast
    enhancement with gadolinium will greatly
    enhance the sensitivity of MRI. Epidural mapping
    via the injection of radiopaque contrast
    under fluoroscopy through a catheter inserted
    through the caudal canal is sometimes used to
    determine the location of epidural scar tissue in
    FBSS patients, often as a precursor to epidural
    lysis of adhesions (ie, Racz procedure) or
    epiduroscopy. However, this procedure provides
    very little additional information. In the patient with implanted hardware, foreign ferromagnetic
    metal objects give rise to local distortion
    of the magnetic field, which can greatly
    degrade MRI results. When implants are made
    of non-superparamagnetic materials like titanium,
    MRI distortion is less but the anatomy
    may still be obscured. Since this patient did not
    have hardware implanted, this should not deter
    the use of MRI. Generally, T2-weighted images
    are more sensitive for detecting pathology,
    whereas T1-weighted images are better for discerning
    anatomy. The use of MRI to follow a
    stable, pathologic condition of the lumbar spine
    is controversial. The use of MRI to evaluate a
    patient with chronic low back pain who has
    recently undergone spine surgery and presents
    with new symptoms is justified
How well did you know this?
1
Not at all
2
3
4
5
Perfectly
18
Q
269. Which of the following is false regarding discogenic
low back pain?
(A) Sitting bent forward subjects the
intervertebral disc to a greater amount
of pressure than lying down, standing
or sitting with one’s back straight
(B) It is often diagnosed by using
provocative discography
(C) Because of their caudad position in the
spine, the lower lumbar discs are most
prone to degenerative disc disease (DDD)
(D) Studies have shown a genetic
predisposition to DDD
(E) Intradiscal steroids are an effective
means for treating DDD
A
  1. (E) Sitting bent forward subjects the lumbar
    intervertebral discs to greater stress than standing,
    sitting with one’s back straight, or lying
    down. This helps explain why patients with
    discogenic low back pain often present with
    sitting intolerance. Although controversial,
    discography, with or without CT scanning, is
    still commonly used to diagnose discogenic
    pain. Patients at high risk for false-positive
    discography include those with psychopathology
    and previous back surgery. The lower
    lumbar discs are more likely to develop degenerative
    changes, and hence become pain generators,
    than more cephalad discs because of
    the increased load they bear. Recent studies
    have shown a genetic predisposition for both
    degenerative disc disease and sciatica. Several
    prospective studies have been conducted evaluating
    intradiscal steroids in patients with
    discogenic low back pain, and none have found
    them to be efficacious.
How well did you know this?
1
Not at all
2
3
4
5
Perfectly
19
Q
  1. Which of the following statements concerning
    central pain is true?
    (A) Spinal cord injury is the leading cause of
    central pain in the United States
    (B) Lesions involving spinothalamocortical
    pathways are necessary and sufficient to
    cause central pain
    (C) Central pain is a common sequelae
    following neurosurgic procedures
    (D) Motor cortex stimulation is an effective
    means to treat central pain
    (E) The most typical presentation of central
    pain is a spontaneous, burning sensation
    on the entire body contralateral to
    the lesion site
A
  1. (D) Owing to its high incidence, stroke is the
    leading cause of central pain in the industrialized
    world. The chance of developing central
    pain following spinal cord injury is higher than
    after stroke (30%-50% vs 8%), but the overall
    number of stroke patients with central pain is
    higher. Syringomyelia is the disorder with the
    highest incidence of central pain (60%-80%).
    According to neurosurgical studies conducted
    by V. Cassinari and C.A. Pagni in the 1960s,
    injury to spinothalamocortical pathways is necessary
    but not sufficient to cause central pain.
    The reason why some patients develop central
    pain but others with identical injuries do not is
    unknown. Central pain may occur after neurosurgical
    procedures and intracranial bleeds,
    but these are unusual occurrences. There are
    now several prospective studies showing
    motor cortex stimulation to be an effective
    treatment for central pain. There is no typical
    presentation for central pain. While spontaneous
    pain is almost universal, allodynia also
    affects a majority of central pain patients. The
    time lag between the injury and onset of pain,
    and the location of central pain are extremely
    variable.
How well did you know this?
1
Not at all
2
3
4
5
Perfectly
20
Q
  1. Which of the following is not commonly used
    to diagnose the level of nerve root involvement
    in radicular pain?
    (A) MRI
    (B) CT scan
    (C) Selective nerve root block
    (D) Electromyography (EMG)/nerve
    conduction studies (NCS)
    (E) Epidural injections with local anesthetic
    and steroids
A
  1. (E) MRI is usually the first test used to evaluate
    new-onset radicular pain. CT scan is less sensitive
    than MRI for detecting disc pathology, but
    is used in patients with pacemakers, spinal
    hardware (owing to the poor resolution of MRI
    in patients with ferromagnetic metal objects)
    and when MRI is not available. Selective nerve
    blocks are sometimes used to diagnose nerve
    root pathology prior to surgery, but there is little
    evidence as to whether or not this improves
    outcomes. Although the terms are sometimes
    used interchangeably, selective nerve blocks are
    not the same as transforaminal epidural injections.
    Since transforaminal epidural injections
    typically result in injectate spread to contiguous
    spinal levels, they cannot be considered diagnostic.
    In addition to providing information
    about the site of nerve root lesions, EMG/NCS
    can help determine whether or not the lesion is
    axonal or demyelinating; whether it is focal,
    multifocal or diffuse; and the age, severity, and
    prognosis of the lesion. QST is a subjective test
    used to evaluate large and small fiber neuronal
    dysfunction. It may be helpful in clarifying
    mechanisms of pain, diagnoses, and guiding
    treatment. It is not used to diagnose nerve root
    pathology.
How well did you know this?
1
Not at all
2
3
4
5
Perfectly
21
Q
272. Which of the following conditions is not generally
associated with a painful neuropathy?
(A) Chronic renal failure
(B) Celiac disease
(C) AIDS
(D) Fabry disease
(E) Amyloidosis
A
  1. (A) Chronic renal failure is associated with
    large, myelinated fiber loss that is rarely
    painful. Celiac disease is a chronic inflammatory
    enteropathy resulting from sensitivity to
    gluten. Neurologic complications are estimated to occur in approximately 10% of patients with
    peripheral neuropathy and ataxia being the
    most common. The neuropathy is usually sensory,
    although infrequently motor weakness
    may develop. There is some evidence that the
    neurologic symptoms associated with celiac
    disease may be ameliorated by a gluten-free
    diet. Peripheral neuropathies are reported to
    affect up to 35% of AIDS patients, being more
    common in later stages of the disease. The most
    common neuropathy in AIDS patients is a
    distal sensory polyneuropathy caused by the
    human immunodeficiency virus (HIV). Other
    causes of neuropathy in AIDS patients include
    toxic neuropathies from medications, co-infection
    with cytomegalovirus (CMV) and other organisms,
    and vitamin B12 deficiency. Fabry disease
    is an X-linked, lysosomal storage disease that
    involves the accumulation of galactosylglucosylceramide
    because of deficiency of α-galactosidase
    A. It usually presents in adulthood; if
    symptoms occur in childhood they usually take
    the form of a painful neuropathy. Amyloidosis
    may result in a painful peripheral or autonomic
    neuropathy secondary to deposition of amyloid
    in nervous tissue. In one study, 35% of
    patients with amyloidosis were found to have
    peripheral neuropathy.
How well did you know this?
1
Not at all
2
3
4
5
Perfectly
22
Q
  1. Apreviously healthy 31-year-old woman presents
    to her internist with generalized muscle
    pain, most prominent in her right thigh. The pain
    travels down the back of her leg to the bottom
    of her foot. She also notes progressive numbness
    and weakness in her arms and legs.
    Walking is difficult and a loss of fine motor
    control makes routine tasks like eating a challenge.
    A review of her medical record reveals
    an URI 3 weeks earlier. Which of the following
    is the most likely diagnosis?
    (A) Multiple sclerosis
    (B) Guillain-Barré syndrome
    (C) Chronic fatigue syndrome
    (D) Acute lumbar and cervical
    radiculopathies
    (E) Diabetic neuropathy
A
  1. (B) The patient’s symptoms are most consistent
    with Guillain-Barré (GB) acute inflammatory
    demyelinating polyneuropathy. Patients
    with GB syndrome generally present with diffuse
    muscular or radicular pain followed by
    sensorimotor dysfunction. Most, but not all
    (72%) patients with GB syndrome experience
    pain during the course of their illness. GB syndrome
    affects 1 to 1.5 people per 100,000 and
    shows no age or gender preference. About 60%
    to 70% of cases are preceded by an URI or gastrointestinal
    (GI) illness 1 to 3 weeks before
    symptoms begin. Cerebrospinal fluid (CSF)
    analysis reveals normal pressures, increased
    protein and no cells. The pathology of GB syndrome
    is demyelination, with most patients fully
    recovering. Multiple sclerosis is a demyelinating
    disease that typically presents in early adult
    life. The most common presenting symptom
    of multiple sclerosis is ocular complaints,
    which affects most patients at some time during
    the course of their illness. Spinal cord lesions
    can produce a myriad of sensorimotor problems
    including weakness, spasticity, hyperreflexia,
    bladder dysfunction, sensory loss, and
    diminished temperature sensation and proprioception.
    Central dysesthetic pain affects
    approximately 20% of multiple sclerosis
    patients. The diagnosis of multiple sclerosis is
    usually supported by MRI, with or without
    CSF analysis. Although muscle pain and weakness
    may be present in chronic fatigue syndrome
    (CFS), the hallmark of this disorder is
    disabling physical and mental fatigue present
    for more than 6 months. There is no firm data
    causally linking viral infection to CFS despite
    frequent reported associations. The most
    common presentation of acute radiculopathy is
    pain or sensory changes in a lower extremity.
    The most common form of diabetic neuropathy
    is distal, symmetrical polyneuropathy. It is predominantly
    a sensory disturbance, occurring
    in a stocking-glove distribution. Because the
    feet are innervated by the longest nerves in the
    body, they are usually the first part of the body
    to be affected. Other types of neuropathy that
    may be present in diabetics include lower
    extremity proximal motor neuropathy, truncal
    neuropathy, cranial mononeuropathy, and autonomic
    neuropathy. The cause of diabetic neuropathy
    is most likely related to metabolic and
    ischemic nerve injury
How well did you know this?
1
Not at all
2
3
4
5
Perfectly
23
Q
  1. Which of the following statements is true
    regarding SI joint pain?
    (A) The SI joint is a diarthrodial synovial
    joint designed primarily for stability
    (B) Patrick’s and Gaenslen’s tests are definitive
    diagnostic tests for SI joint pain
    (C) CT scanning is the most sensitive means
    for diagnosing SI joint pain
    (D) Lifting heavy objects is the one of the
    most common causes of SI joint injury
    (E) When diagnostic blocks fail, surgery can
    usually provide long-term pain relief
A
  1. (A) The SI joints are large, paired, diarthrodial
    synovial joints whose primary functions are
    stability and dissipating truncal loads. The
    joints are also involved in limiting x-axis rotation
    and in women, parturition. There are literally
    dozens of provocative tests that have
    been advocated as screening tools for SI joint
    pain, but several studies have shown that these
    tests lack both specificity and high sensitivity.
    On a similar note, CT scanning may show SI
    joint pathology in over 30% of asymptomatic
    control patients, and be negative in over 40% of
    patients with SI pain. The most reliable method
    for diagnosing SI joint pain is through diagnostic
    local anesthetic blocks. The mechanism
    of SI joint injury has been described as a combination
    of axial loading and sudden rotation.
    Common causes of SI joint pain include motor vehicle accidents, falls, athletic injuries, spondyloarthropathies,
    and pregnancy. SI joint injections
    with corticosteroids have been shown in
    some but not all studies to provide short-term
    pain relief. SI joint pain is usually not amenable
    to surgical correction.
How well did you know this?
1
Not at all
2
3
4
5
Perfectly
24
Q
275. Which of the following statements regarding
headaches is false?
(A) The International Headache Society’s
diagnostic criteria for cervicogenic
headaches includes unilaterality of
symptoms and relief of pain by
diagnostic anesthetic blocks
(B) Migraine with aura is more common
than migraine without aura
(C) In chronic tension-type headache, the
average headache frequency is equal to
or greater than 15 days per month
(D) Cluster headaches are more prevalent in
men than in women
(E) Tricyclic antidepressants are a mainstay
of treatment for both migraine and
tension-type headaches
A
  1. (B) In population-based studies, migraine without
    aura is about twice as frequent as migraine
    with aura. Major criteria for the diagnosis of
    cervicogenic headache include signs and symptoms
    of neck involvement such as the precipitation
    of head pain by neck movement or
    external pressure over the upper cervical or
    occipital region, restricted range of motion in
    the neck, unilaterality of head pain with or
    without shoulder or arm pain, and confirmatory
    evidence by diagnostic anesthetic blocks.
    Chronic tension-type headache differs from
    episodic tension-type headache in that the
    average headache frequency is equal to or
    greater than 15 days per month or 180 days
    per year. A shift from peripheral to central
    mechanisms is believed to play a role in the
    evolution of episodic to chronic tension-type
    headache. Cluster headaches typically present
    as a series of intense unilateral headaches
    occurring over a period of 2 weeks to 3 months.
    They are associated with unilateral autonomic
    features such as nasal congestion, rhinorrhea,
    miosis, or lacrimation. The attacks are usually
    brief, lasting between 15 and 180 minutes, and
    occur in the orbital, supraorbital and/or temporal
    regions. Unlike migraine headaches,
    tension-type headaches, temporal arteritis, and
    cervicogenic headaches, cluster headaches are
    more frequent in men, with an average male to
    female ratio of 5 to 1. Tricyclic antidepressants
    have been shown in numerous clinical trials to
    be effective in the prevention of both migraine
    and tension-type headaches.
How well did you know this?
1
Not at all
2
3
4
5
Perfectly
25
Q
  1. Which of the following statements regarding
    postamputation pain is correct?
    (A) Vascular conditions are the leading
    cause of both lower and upper
    extremity amputations
    (B) There is no relationship between
    persistent stump pain and phantom
    limb pain in amputees
    (C) The intensity of pain and the length of
    the phantom increases with time
    (D) Phantom breast pain is a common cause
    of postmastectomy pain
    (E) Phantom pain was first described in the
    American Civil War
A
  1. (D) Phantom breast pain occurs in roughly 20%
    of mastectomy patients, and phantom sensations
    in close to half. Originally thought to be
    rare, phantom limb pain is now recognized to
    occur in between 60% and 80% of limb
    amputees. Phantom limb pain must be distinguished
    from phantom sensations, which occur
    in over 90% of patients. Vascular conditions
    account for over 80% of limb amputations in
    the United States. However, trauma is responsible
    for approximately 75% of upper extremity
    amputations. Most researchers have found
    a statistically significant association between
    phantom limb pain and persistent stump pain.
    Although earlier studies found a correlation
    between preamputation pain and phantom
    limb pain, more recent studies have not confirmed
    this relationship. It is widely held that
    phantom pain diminishes with time and eventually
    fades away. Though described, phantom
    pain associated with congenital absence of a
    limb is rare. Phantom pain is generally worse in
    the distal part of a limb. Most phantoms shrink
    with time, with the most distal aspect of a limb
    being the last to disappear. This is known as
    “telescoping,” and occurs in approximately
    half of all limb amputees. Archaeological
    records demonstrate that purposeful amputations
    have been practiced since Neolithic times.
    The concept of “phantom pain” has been recognized
    for hundreds, if not thousands, of
    years. In the 16th century, the French military
    surgeon Ambrose Paré outlined clear distinctions
    between phantom limb pain, phantom
    sensation, and stump pain. The term “phantom
    pain” was coined by Weir Mitchell in the
    American Civil War. A few years earlier,
    Mitchell used the word “causalgia” to describe
    the characteristic autonomic changes found in
    the extremities of soldiers who suffered major
    nerve damage.
How well did you know this?
1
Not at all
2
3
4
5
Perfectly
26
Q
  1. Which of the following statements regarding
    the assessment of pain in pediatric patients is
    true?
    (A) Palmar sweating and reduced transcutaneous
    oxygen concentrations are indicative
    of pain
    (B) In a hospitalized 2-year-old child, crying
    and increased vitals signs are likely to
    indicate chronic pain
    (C) The FACES scale and Charleston Pain
    Pictures provide accurate assessments
    of pain in preschool children
    (D) The COMFORT scale and facial action
    coding system (FACS) are pain instruments
    used in young children that are
    based predominantly on facial actions
    (E) Visual analogue and numerical rating
    scales are inappropriate pain indices for
    most adolescents
A
  1. (A) Palmar sweating and reduced transcutaneous
    oxygen concentration are indicative of,
    though not specific for, acute pain. In a young
    child, crying and increased vital signs (eg. heart
    rate, respiratory rate, and blood pressure) are
    associated with distress, which includes but is
    not limited to pain. Other factors that may
    cause these signs include separation anxiety,
    hunger, and fear. Unlike acute pain, chronic
    pain is usually not associated with elevated
    vital signs. The FACES scale and Charleston
    Pain Pictures are designed to provide assessments
    of pain in school aged, not preschool
    children. The FACS and COMFORT scale are
    used to assess pain in infants and young children.
    The FACS is a comprehensive coding system based on a wide range of facial actions.
    The COMFORT scale is an eight-item scale
    designed to measure distress (including pain)
    that includes alertness, calmness, respiratory
    response, physical movement, blood pressure,
    muscle tone, and facial tension. Pain scales
    used in adults such as verbal pain scores,
    numerical rating scales, and visual analogue
    scales provide accurate assessments of pain in
    most adolescents.
How well did you know this?
1
Not at all
2
3
4
5
Perfectly
27
Q
  1. Which of the following statements is not correct
    regarding herpes varicella zoster?
    (A) The most common presentation of acute
    herpes zoster (AHZ) is pain and a vesicular
    rash in the midthoracic dermatomes
    (B) The polymerase chain reaction (PCR) is
    the most common means to diagnose
    AHZ
    (C) The incidence of both AHZ and postherpetic
    neuralgia increases with age
    (D) There is no generally accepted time
    period from the onset of AHZ to when a
    diagnosis of postherpetic neuralgia is
    made
    (E) AHZ involving the lumbosacral
    dermatomes may be misdiagnosed as a
    herniated disc
A
  1. (B) The most common way to diagnose AHZ is
    clinically. In a small percentage of patients,
    AHZ may occur without a rash, a condition
    known as “zoster sine herpete” (zoster without
    rash). The PCR is often used to aid in the diagnosis
    of this condition. In descending order,
    the most common sites for AHZ are the
    midthoracic dermatomes, the ophthalmic division
    of the trigeminal nerve, and the cervical
    region. The incidence of both AHZ and postherpetic
    neuralgia increases with age. Other
    risk factors for AHZ include HIV infection and
    transplant surgery, which is likely because of
    the resultant immunosuppression. There is no
    standard time period after which persistent
    pain from AHZ is diagnoses as postherpetic
    neuralgia. Postherpetic neuralgia has been variably
    defined as the persistence of sensory
    symptoms 1 month, 6 weeks, 2 months, 3 months,
    and 6 months after herpes zoster. AHZ affects
    the lumbosacral dermatomes in between 5%
    and 15% of patients. Lumbosacral AHZ may be
    misdiagnosed as a herniated disc
How well did you know this?
1
Not at all
2
3
4
5
Perfectly
28
Q
  1. Which of the following statements regarding
    electrophysiologic testing is true?
    (A) Nerve conduction velocities are more
    likely to decrease in conditions such as
    alcoholic and diabetic neuropathy that
    are characterized by Wallerian degeneration
    than in demyelinating neuropathies
    such as Guillain-Barré
    (B) EMG can provide information about the
    type, extent and timing of injuries to
    motor units and individual muscle
    fibers
    (C) The H reflex can aid in the evaluation of
    brachial plexus injuries
    (D) The F response is used to diagnose pure
    sensory neuropathies
    (E) EMG can readily identify processes
    causing muscle denervation (neuropathies),
    but is incapable of identifying
    myopathies
A
  1. (B) EMG provides a wealth of information
    about the integrity, function, and innervation of
    motor units and (using special techniques) individual
    muscle fibers. Serial EMG examinations
    permit monitoring of recovery or disease progression.
    A normal EMG indicates the absence
    of motor unit involvement. In neuropathies
    characterized by Wallerian degeneration, nerve
    conduction velocities range from low normal to
    mildly slow. In contrast, demyelinating neuropathies
    of the acute and chronic inflammatory
    types produce segmental demyelination,
    which markedly slows conduction velocities.
    The H wave is the electrical representation of the
    tendon reflex circuit. In adults, it is only obtainable
    in the lower reflexes. It is most prominent
    during stimulation of the tibial nerve, being
    particularly helpful in the diagnosis of S1
    radiculopathy and predominantly sensory
    polyneuropathies. The F wave is a late response
    that is evoked by supramaximal stimulation of
    a motor nerve. It occurs when a small percentage
    of the stimulated motor neurons “rebound.”
    The initial response to stimulation of a motor
    nerve is the M wave. Unlike H waves, F waves
    are not true reflexes.
How well did you know this?
1
Not at all
2
3
4
5
Perfectly
29
Q
  1. Which of the following statements is true about
    quantitative sensory testing (QST)?
    (A) QST can be used to pinpoint which
    nerve is injured and where along its
    path the lesion lies
    (B) Thermal sensation is used to measure
    the integrity of large, myelinated nerve
    fibers
    (C) A beta function can be evaluated using
    either a tuning fork or von Frey hair
    (D) QST can be used to evaluate the function
    of all different types of nerve fibers
    (E) An advantage of QST is that it can accurately
    assess function in uncooperative
    or incapacitated patients
A
  1. (C) Large, myelinated nerves are more vulnerable
    to injury than small neurons. The function
    of large, myelinated A-β function can be
    measured using both vibratory thresholds and
    von Frey filaments. QST is used to evaluate the
    function of individual nerve fibers. It is not
    useful in determining which nerve is injured
    and where along its path the injury lies. Both
    cold and hot thermal sensations are used to
    measure the function of small myelinated (A-δ)
    and unmyelinated C fibers. QST cannot be
    used to assess B (preganglionic autonomic) and
    A-γ (muscle spindle efferent) function. Adownside
    of QST is that its accuracy is dependent on
    the cooperation and reliability of the patient.
How well did you know this?
1
Not at all
2
3
4
5
Perfectly
30
Q
  1. A 38-year-old construction worker presents to
    you with complaints of right lower extremity
    pain for the last 8 months. Pain radiates from
    the lower back to the outer aspect of the right
    leg and goes down to the dorsum of the right
    foot. The patient reports a problem with walking
    and on examination reveals an antalgic gait
    and inability to do heel-walking on right,
    though toe-walking is not affected. Strength is
    5/5 in all muscle groups except dorsi-flexion of
    the right ankle which is 4/5 and strength testing
    for extensor hallucis longus reveals 4/5
    strength. Deep tendon reflexes are 2+ at both
    knees and both ankles. Sensory testing reveals
    mildly reduced sensation to light touch and
    pinprick on the dorsum of the right foot when
    compared to the left foot. This patient most
    likely has
    (A) right piriformis syndrome
    (B) right L4 radiculopathy
    (C) right L5 radiculopathy
    (D) right S1 radiculopathy
    (E) facet arthritis
A
  1. (C) Lumbar radiculopathy most often results
    from disc herniation. Depending on the level of
    herniated discs radiculopathy may affect specific
    nerve roots. Disc herniation at L4-5 and
    L5-S1 is most likely caused by mobility of the
    segment. A herniated disc may compromise
    the nerve root at the same level if displaced
    laterally in the recess or in the foramen (L4-5
    disc affecting L4 nerve root), or it may effect the
    traversing nerve root to the level below (L4-5
    disc affecting L5 nerve root). L5 radiculopathy
    results in pain, sensory, and motor changes in
    L5 dermatomal distribution. Pain is usually
    described as shooting or occasionally aching
    and burning sensation on the outside of leg
    radiating to the dorsum of foot. Sensory testing
    may also reveal a decrease in light-touch and
    pinprick sensation in the same distribution.
    L5 radiculopathy also may result in weakness
    in the extensor hallucis longus and thus heel
    walking. Deep tendon reflexes may be spared
    in the lower extremity.
How well did you know this?
1
Not at all
2
3
4
5
Perfectly
31
Q
  1. A46-year-old female with past medical history
    of depression, anxiety, irritable bowl syndrome,
    and asthma is referred to you for evaluation of
    her lower back pain. History reveals onset of
    generalized pain that started after she was
    involved in a car accident 4 years ago. Physical
    examination reveals nonfocal neurologic
    examination. Musculoskeletal examination
    reveals multiple areas of hypersensitivity. The
    patient reports marked pain with moderate
    digital pressure at base of skull, her neck, front
    of her chest, her elbows as well as her lower
    back, and bilateral lower extremities. The
    patients MRI scan of the lumbar spine reveals
    preserved disc height, no facet arthritis and
    minimal disc bulge at L4-5 without any spinal
    or foraminal stenosis. This patient most
    likely has
    (A) fibromyalgia syndrome
    (B) discogenic pain
    (C) myofascial pain disorder
    (D) somatoform disorder
    (E) opioid hyperalgesia
A
  1. (A) Fibromyalgia syndrome is a common pain
    condition, estimated to occur in 2.4% of the
    general population. The syndrome is characterized
    by widespread musculoskeletal pain,
    sleep disturbance, psychologic distress, and
    comorbidity with other pain syndromes [eg,
    irritable bowel syndrome (IBS), interstitial cystitis,
    and the female urethral syndrome], which
    have considerable impact on the everyday life
    of patients. Fibromyalgia syndrome occurs predominantly
    in women and demonstrates familial
    aggregation. Since 1990, the diagnosis of
    fibromyalgia syndrome has been based on criteria
    of the American College of Rheumatology
    (ACR). A key dimension of the ACR criteria is
    the concept of tender points, 18 specific points
    on the body surface at which digital palpation
    elicits pain (11/18 “positive” tender points fulfills
    an fibromyalgia criteria).
    It is not uncommon for patients to have
    other pain pathologies in addition to fibromyalgia.
    However a complete clinical picture should
    be viewed before consideration of treatment
    options especially if it involves interventional
    procedures. Patient describes above most likely
    has fibromyalgia as evidenced by the presence
    of tender points. A negative physical examination
    except for tender points and hypersensitivity
    argues against other listed options.
How well did you know this?
1
Not at all
2
3
4
5
Perfectly
32
Q
  1. A25-year-old, healthy female volleyball player
    has developed severe pain in right hand. This
    pain started while playing volleyball and after
    a reported wrist sprain. One month after the
    initial injury and despite conservative care with
    nonsteroidal anti-inflammatory drugs (NSAIDs),
    muscle relaxants, and hand splint to avoid any
    movement related pain, the patient complains
    of even worse burning pain. Pain is worse with
    light touch, even blowing air or rubbing of
    clothes trigger unbearable pain. The patient
    also reports her right hand to be cold and often
    wet because of localized sweating. On examination
    the patient has a markedly swollen, redappearing
    hand. Patient is unable to make a
    fist with her fingers and measurement of temperature
    reveals a 7°C lower temperature compared
    to opposite extremity. Which of the
    following is the most likely diagnosis?
    (A) CRPS I (RSD)
    (B) CRPS II (causalgia)
    (C) Peripheral vascular disease
    (D) Deep venous thrombosis of upper
    extremity
    (E) Median neuralgia
A
  1. (A) Following is the diagnostic criteria for
    CRPS I:
  2. The presence of an initiating noxious event
    or a cause of immobilization.
  3. Continuing pain, allodynia, or hyperalgesia
    with which the pain is disproportionate
    to any inciting event.
  4. Evidence at some time of edema, changes
    in skin blood flow, or abnormal sudomotor
    activity in the region of the pain.
  5. This diagnosis is excluded by the existence
    of condition that otherwise would account
    for the degree of pain and dysfunction.
    The patient in the question meets all the
    criteria for diagnosis of CRPS I (RSD). CRPS II
    (causalgia) by definition has a known injury to
    a major nerve. Vascular etiology though possible
    after trauma, is unlikely to give symptoms
    of allodynia as well as sudomotor changes.
    Median neuralgia would result in a similar
    clinical pain picture but only hand discomfort
    would be expected to be confined only to the
    distribution of median nerve.
How well did you know this?
1
Not at all
2
3
4
5
Perfectly
33
Q
  1. A 38-year-old man developed complete T4
    spinal cord injury after a motorcycle accident.
    Two months after the injury the patient continues
    to complain of severe radiating pain to the front of chest just above nipple line. The pain is
    worse with light touching and improves with
    movement restriction and use of morphine on as
    needed basis. This patient most likely has
    (A) central dysesthesia syndrome
    (B) syringomyelia
    (C) transitional zone pain
    (D) myofascial pain
    (E) autonomic dysreflexia
A
  1. (C) Spinal cord injury may result in various
    types of pain. To provide the most effective
    treatment—understanding the mechanism of
    pain is very important. Taxonomy of spinal
    cord injury pain may be divided into neuropathic
    or nociceptive pain. The patient in question
    appears to have most likely nerve root
    impingement at T4-5, level of his spinal cord
    injury, resulting in severe T4 neuralgic pain
    radiating towards the front of chest wall.
34
Q
  1. After a car accident 5 days ago, a 42-year-old
    engineer reports severe neck and midback
    pain. The patient was rear ended while stopped
    at a traffic light by a pickup truck. The patient
    reports severe pain in neck that radiates down
    to both shoulders and upper arm as well as to
    the midback region. The pain is a severe stabbing
    and aching sensation that is markedly
    exaggerated by movement of neck. Examination
    reveals otherwise intact neurologic system,
    5/5 strength, and intact deep tendon reflexes
    without any sensory deficit. Imaging studies
    are essentially normal except for straightening
    of cervical lordosis. The patient most
    likely has
    (A) bilateral C5 radiculopathy
    (B) myofascial pain
    (C) fibromyalgia
    (D) thoracic outlet syndrome
    (E) malingering
A
  1. (B) Myofascial pain may result after a sudden
    acceleration-deceleration insult. Neck muscles
    may reflexly go into spasm. It may also result
    in straightening of cervical lordosis secondary
    to spasm of posterior supporting neck muscles.
    Myofascial pain from cervical neck muscle may
    radiate between shoulder blades as well into
    the upper extremity. Negative imaging studies
    are essential to rule out traumatic disc herniation
    or fracture. Treatment includes nonsteroidals,
    muscle relaxants, and physical therapy.
    In a small percentage of patients, if pain doesn’t
    resolve trigger point injections or cervical
    medial branch blocks may provide help with
    continuing physical therapy.
35
Q
  1. A 64-year-old female with a history of coronary
    artery disease, peripheral vascular disease,
    and type 1 diabetes mellitus, controlled
    with insulin, presents to your pain clinic with
    gradually worsening bilateral leg and feet pain.
    The patient reports a history of a fall approximately
    5 years ago which resulted in severe
    back and leg pain. That pain resolved, however,
    the patient started developing numbness
    and tingling in both legs and feet. On examination
    the patient reveals otherwise normal
    appearing legs and feet, patient does have a
    nonhealing ulcer on her right great toe.
    Neurologic testing reveals bilateral 5/5 muscle
    strength and 2+ patellar and ankle reflexes.
    Sensory testing reveals intact proprioception
    but reduced sensation to light touch and
    pinprick. The patient also reported marked
    sensitivity to light touch. This patient most
    likely has
    (A) CRPS I
    (B) peripheral vascular disease
    (C) diabetic polyneuropathy
    (D) lumbar spondylosis
    (E) central pain
A
  1. (C) In type 1 diabetes mellitus, distal polyneuropathy
    typically occurs after many years of
    chronic prolonged hyperglycemia. Conversely,
    in type 2, it may present after only a few years
    of poor glycemic control. Occasionally, in type 2,
    diabetic neuropathy is found at the time of
    diagnosis (or even predating diagnosis).
    Diabetic neuropathy can manifest with a
    wide variety of sensory, motor, and autonomic
    symptoms. Sensory symptoms may be negative
    or positive, diffuse or focal. Negative sensory
    symptoms include numbness; “deadness”; feeling
    of wearing gloves or walking on stilts; loss
    of balance, especially with the eyes closed; and
    painless injuries. Positive symptoms include
    burning, pricking pain, electric shocklike feelings,
    tightness, and hypersensitivity to touch. Motor symptoms can cause distal, proximal,
    or focal weakness. Autonomic symptoms may
    be sudomotor, pupillary, cardiovascular, urinary,
    GI, and sexual.
    A generally accepted classification of diabetic
    neuropathies divides them broadly into
    symmetric and asymmetric neuropathies.
    Symmetric polyneuropathies involve multiple
    nerves diffusely and symmetrically and are
    the most common form. The patient in question
    appears to have symmetrical small and
    large fiber neuropathy resulting in pain in
    both legs and feet, and decreased light-touch
    sensation as well as allodynia.
36
Q
  1. A32-year-old female develops severe stabbing,
    “like an ice pick,” pain at the base of tongue
    after an infratemporal neurosurgic procedure.
    Pain comes in paroxysms and last a few seconds
    and is triggered by swallowing, yawning,
    and coughing. This patient most likely has
    (A) trigeminal neuralgia
    (B) geniculate neuralgia
    (C) glossopharyngeal neuralgia
    (D) migraine with atypical aura
    (E) cluster headache
A
  1. (C) Glossopharyngeal neuralgia is a disorder
    characterized by intense pain in the tonsils,
    middle ear, and back of the tongue. The pain
    can be intermittent or relatively persistent.
    Swallowing, chewing, talking, sneezing, or
    eating spicy foods may trigger the disorder. It
    is often the result of compression of the 9th
    nerve (glossopharyngeal) or 10th nerve (vagus),
    but in some cases, no cause is evident.
    Skull base surgery or surgeries in the
    infratemporal region may result in damage
    or irritation of glossopharyngeal nerve.
    Conservative treatment includes using anticonvulsants.
    In refractory cases glossopharyngeal
    nerve block may be helpful. Radiofrequency
    lesioning or neurolytic treatment should be
    reserved for resistant cases or ones associated
    with head and neck cancer. Surgical decompression
    should be reserved for nonresponders
    and resistant cases.
37
Q
  1. A38-year-old patient care technician while lifting
    a 400 lb patient heard a pop in his back. The
    patient developed severe back pain with radiation
    to the right leg. Patient described the pain
    as stabbing back pain with electrical sensations
    down the back of the right leg all the way to the
    sole of the right foot. On examination the
    patient appeared very uncomfortable, sitting
    in a wheel chair. Straight leg raise and cross
    straight leg raise test was positive. Muscle
    strength was 5/5 in all muscle groups except
    plantar flexion at right ankle which was 4/5.
    Deep tendon reflexes were intact at the patella
    bilaterally; however, the reflex at the right ankle
    is diminished compared to the left ankle. The
    patient most likely has a herniated disc at
    (A) L4-L5 resulting in L4 nerve root
    compression
    (B) L4-L5 resulting in L5 nerve root
    compression
    (C) L5-S1 resulting in L5 nerve root
    compression
    (D) L5-S1 resulting in S1 nerve root
    compression
    (E) L1-L2 resulting in compression of cauda
    equina
A
  1. (D) Lumbar radiculopathy most often results
    from disc herniation. Depending on the level
    and “direction” of herniated discs a resultant
    radiculopathy may affect specific nerve roots.
    Disc herniation at L5-S1 is most likely a result
    of mobility of the segment. A herniated disc
    may compromise the nerve root at the same
    level if displaced laterally in the recess or in the
    foramen (L5-S1 disc affecting L5 nerve root), or
    it may affect the traversing nerve root to the
    level below (L5-S1 disc affecting S1 nerve root).
    S1 radiculopathy results in pain, sensory, and
    motor changes in S1 dermatomal distribution.
    Pain is usually described as shooting or
    occasionally as an aching and burning sensation
    on the back of thigh radiating to the plantar
    aspect (sole) of foot. Sensory testing may
    also reveal a decrease in light-touch and pinprick
    sensation in the same distribution. S1
    radiculopathy also may result in weakness in
    Plantar flexion and thus toe walking. Most
    often with significant S1 nerve root compression,
    ankle reflex is diminished. Examination
    also may reveal positive straight leg raise and
    cross straight leg raise test (reproduction of
    radiating pain in lower extremity by raising
    the opposite extremity).
38
Q
  1. A48-year-old patient after a gunshot wound to
    the upper chest develops a partial cord transection
    involving the right spinothalamic tract
    at T2 level. This patient is most likely to develop
    loss of pain and temperature sensation:
    (A) At the level of the transection
    (B) Below and on right side from the level
    of transection
    (C) Below and on left side from the level of
    transection
    (D) Patient is not likely to develop central
    dysesthetic pain
    (E) Below and bilateral lower extremity
A
  1. (C) The spinal cord is organized into a series of
    tracts or neuropathways that carry motor
    (descending) and sensory (ascending) information.
    These tracts are organized anatomically
    within the spinal cord. The corticospinal
    tracts are descending motor pathways located
    anteriorly within the spinal cord. Axons extend
    from the cerebral cortex in the brain as far as
    the corresponding segment, where they form
    synapses with motor neurons in the anterior
    (ventral) horn. They decussate (cross over) in
    the medulla prior to entering the spinal cord.
    The dorsal columns are ascending sensory
    tracts that transmit light-touch, proprioception,
    and vibration information to the sensory
    cortex. They do not decussate until they reach
    the medulla. The lateral spinothalamic tracts
    transmit pain and temperature sensation.
    These tracts usually decussate within three
    segments of their origin as they ascend. The
    anterior spinothalamic tract transmits light
    touch. Autonomic function traverses within
    the anterior anteromedial tract. Sympathetic
    nervous system fibers exit the spinal cord
    between C7 and L1, while parasympathetic
    system pathways exit between S2 and S4.
    Injury to the corticospinal tract or dorsal
    columns, respectively, results in ipsilateral
    paralysis or loss of sensation of light touch,
    proprioception, and vibration. Unlike injuries
    of the other tracts, injury to the lateral
    spinothalamic tract causes contralateral loss
    of pain and temperature sensation two to three
    segments below the level of injury. Because
    the anterior spinothalamic tract also transmits
    light-touch information, injury to the dorsal columns may result in complete loss of vibration
    sensation and proprioception but only
    partial loss of light-touch sensation. Anterior
    cord injury causes paralysis and incomplete
    loss of light-touch sensation.
39
Q
  1. A38-year-old police officer reports continuous
    neck pain lasting past 6 months. The patient
    recalls lifting and carrying heavy boxes while
    moving his house and reports some neck pain
    at that time. Pain has gradually worsened over
    the past 6 months and now patient reports
    heaviness and occasional weakness in his right
    hand. The patient often feels numbness in right
    index finger as well. On examination, the
    patient has 5/5 strength in all muscle groups
    except mild weakness in flexors of the right
    elbow. Light-touch sensation is intact in all
    dermatomes, however, the patient reports
    increased sensation to light touch in the radial
    aspect of the right forearm. Deep tendon
    reflexes are intact bilaterally except for right
    brachioradialis reflex which is 1+ compared to
    left. This patient most likely has
    (A) right C5 radiculopathy
    (B) right C6 radiculopathy
    (C) right C7 radiculopathy
    (D) right C8 radiculopathy
    (E) cervical facet arthritis with referred pain
A
  1. (B) Patients with a C6 radiculopathy should
    have pain in the neck, shoulder, lateral arm,
    radial forearm, dorsum of hand, and tips of
    thumb, index, and long finger. Distribution of
    pain is less extensive and more proximal,
    whereas paresthesias predominate distally. In
    some individuals, a C6 lesion will manifest as
    a depressed or absent biceps reflex; in others,
    an abnormal brachioradialis or wrist extensor
    reflex can be found. Elbow flexion will be
    weak, and the patient will be unable to
    supinate the forearm against resistance with
    the elbow held in extension. Conservative treatment
    includes physical therapy, traction, and
    analgesics. If pain persists, cervical epidural
    steroid injection may provide relief from pain
    and aid in physical therapy. However, if symptoms
    persist or weakness/numbness doesn’t
    improve surgical decompression with or without
    anterior fusion may be considered
40
Q
  1. A 42-year-old man underwent a celiac plexus
    block procedure with 20 mL of 50% alcohol.
    All of the following listed conditions are complications
    of this intervention EXCEPT
    (A) genitofemoral neuralgia
    (B) hypertension
    (C) diarrhea
    (D) paralysis
    (E) infection
A
  1. (B) Celiac plexus block is both a diagnostic and
    therapeutic tool to help in managing upper
    abdominal pain arising from viscera. Pancreatic
    cancer is the leading diagnosis for neurolytic
    celiac plexus block; other conditions may
    include visceral pain arising from malignancies
    of liver or GI tract.
    The procedure is performed either under
    fluoroscopic guidance or CT scan, though
    blind approaches have also been described.
    Both single transaortic as well bilateral needle
    approaches have been described. The fluoroscopic
    image in question demonstrates a single
    needle transaortic celiac plexus block.
    Complications include diarrhea, hypotension,
    genitofemoral neuralgia, infection, bleeding,
    damage to surrounding structures and rarely
    paralysis. All complications mentioned above
    may occur except hypertension
41
Q
292. Atwo-needle lumbar sympathetic plexus block
at L2 and L3 when performed appropriately
may help in the diagnosis of
(A) sympathetically mediated pain
(B) lumbar discogenic pain
(C) lumbar radiculopathy
(D) diabetic neuropathy
(E) facet arthritis
A
  1. (A)Aproper diagnostic test requires a preblock
    patient evaluation (with special attention to the
    ipsilateral lower extremity pain, temperature,
    and condition), a local anesthetic injection
    using appropriate volume to avoid spread to
    adjacent nerves and a postblock evaluation of
    subjective improvement in pain score as well
    an objective increase in the temperature of the
    involved extremity is crucial. Significant
    improvement in pain scores with increase in
    temperature of the involved extremity points
    toward a positive diagnosis of sympathetically
    mediated pain.
    Discography is performed for diagnosis of
    lumbar discogenic pain. Whereas diabetic neuropathy
    may result in sympathetically mediated
    pain, it is a mixed somatic polyneuropathy and
    diagnosis is a clinical one. Lumbar selective
    nerve root block and facet joint injections may
    aid in the diagnosis of lumbar radiculopathy
    and facet arthritis resulting in pain.
42
Q
  1. A patient who received 1 cc of 0.25% bupivacaine
    after negative aspiration following a cervical
    selective nerve root injection became
    agitated and then developed generalized tonicclonic
    movements. Which of the following is
    the most likely explanation?
    (A) High spinal anesthetic from accidental
    intrathecal injection
    (B) Anxiety attack from pain during
    injection
    (C) Vertebral artery injection of local
    anesthetic
    (D) Injection into spinal cord
    (E) Hypoxia
A
  1. (C) Cervical selective nerve root injection may
    be indicated for diagnosis and treatment of cervical
    radiculopathy. Complication other than
    infection, bleeding, and nerve damage, include
    intravascular uptake into vertebral artery or
    radicular arteries resulting in seizure, stroke, or
    paraplegia. Intraspinal spread into epidural
    or intrathecal spread is also possible resulting
    in high spinal anesthetic. Damage to spinal
    cord has also been reported with injection into
    the spinal cord. Considering the life-threatening
    complications, cervical selective nerve root
    block should only be performed by physicians
    well versed in this technique.
43
Q
294. Medial branch nerve blocks may aid in the
diagnosis of
(A) facet arthritis
(B) sympathetically mediated pain
(C) spinal nerve irritation
(D) sciatica
(E) myofascial pain
A
  1. (A) Medial branches of the dorsal ramus provide
    innervations to the respective facet joint as
    well to the joint below. A diagnostic medial
    branch block with local anesthetic performed at
    appropriate levels (eg, L3 and L4 for L4-5 facet
    joint) may provide diagnostic and prognostic
    information to help with pain associated with
    facet arthritis.
    If pain is considerably albeit transiently
    improved after diagnostic medial branch blocks,
    a radiofrequency ablative procedure may be
    considered to provide longer lasting pain relief.
44
Q
295. Which of the following is the most likely side
effect of a SI joint injection?
(A) Perforation of bladder
(B) Left lower extremity weakness
(C) Stroke
(D) High spinal resulting in cardiorespiratory
depression
(E) Injury to pudendal nerve
A
  1. (B) SI joint injection is performed for both diagnostic
    and therapeutic reasons in patient complaining
    of SI joint pain. After a therapeutic injection with 5 to 10 mL of local anesthetic; it is
    possible that the local anesthetic may spill inferiorly
    and anteriorly and anesthetize sciatic
    nerve resulting in leg weakness. Patients may
    be warned about this, if observed afterward,
    and should be accompanied by a reasonable
    adult to avoid any falls and resultant injuries
45
Q
296. The potential complications of the vertebroplasty
procedure include all EXCEPT
(A) spinal cord compression
(B) venous embolism
(C) pedicle fracture
(D) cement leak in soft tissue
(E) bowl perforation
A
  1. (E) Vertebroplasty is an advanced procedure
    that is performed to stabilize recently fractured
    vertebral bodies resulting in excruciating back
    pain. Performed properly and by trained physicians,
    vertebroplasty is a safe procedure.
    However, complications, though rare, are possible
    and uncompromising. These may include
    infection; bleeding; pulmonary embolus;
    damage to pedicles, spinal cord, or surrounding
    structures; allergic reactions to injectate;
    and cement leak into soft tissue or in spinal
    canal resulting in spinal cord compression.
46
Q
  1. A 70-year-old man reports severe cramps and
    “charley horse” sensation in both legs when
    walking more than one block. Resting usually
    helps in relieving pain. On examination patient
    reveals an intact neurologic examination without
    any sensory or motor deficit. Lower
    extremity examination reveals normal appearance,
    and no vascular insufficiency. Ankle
    brachial index performed 1 month ago is unremarkable.
    Which of the following is the most
    likely diagnosis?
    (A) Neurogenic claudication
    (B) Vascular claudication
    (C) Diabetic peripheral neuropathy
    (D) Amyloid neuropathy
    (E) Fibromyalgia
A
  1. (A) Spinal stenosis may result from narrowing
    of the spinal canal secondary to hypertrophy of
    ligamentum flavum, articular processes and
    anteriorly from degenerative bulging discs.
    Stenosis may result in a classical presentation of
    neurogenic claudication with pain in lower legs
    or feelings of “charley horse” that come with
    walking an unpredictable distance and is
    relieved by resting or sitting down. In contrast
    to vascular claudication, the patient may report
    some back pain as well. In addition, pain is not
    predictably elicited after a certain walking distance
    because it is relative extension of the
    lumbar segments that results in worsening
    stenosis and neurogenic claudication rather
    than ischemia. Pain is relieved in neurogenic
    claudication by assuming a flexion posture
    (bending forward).
    Treatment includes posture education,
    education and improvement of body mechanics,
    and physical therapy. Epidural steroid
    injection series may provide pain relief in
    some patients. If pain or significant limitation
    in activity persists a decompressive laminectomy
    may be considered.
47
Q
  1. A32-year-old healthy female presents with a 2-
    month history of gradually worsening right
    lower extremity pain. The pain is sharp shooting
    in character and radiates down the right leg
    all the way to the right foot. On examination,
    patient has 5/5 muscle strength in all muscle
    groups except plantar flexors of the right ankle.
    The patient is unable to stand on her toes. There
    is no sensory deficit. Flexion, adduction, and
    internal rotation of the right hip results in
    reproduction of the symptoms. MRI of lumbar
    spine is normal with no evidence of herniated
    discs. This patient most likely has
    (A) right S1 radiculopathy
    (B) piriformis syndrome
    (C) SI arthritis
    (D) somatization disorder
    (E) discogenic pain
A
  1. (B) The piriformis is a sausage-shaped muscle
    which originates from the anterior surface of
    the lateral sacrum and attaches to the greater
    trochanter. In most individuals the sciatic nerve
    lies anterior to the muscle belly. Spasm of the
    muscle may result in irritation of the sciatic
    nerve and resultant sciatica. The patient may
    report localized tenderness in the lower part of
    the buttock. In addition, if patients have irritation
    of sciatic nerve, they may also report
    symptoms suggestive of sciatica which may
    easily be confused with lumbar radiculopathy.
    However, flexion, adduction, and internal rotation
    of the thigh results in tightening of piriformis
    muscle which may reproduce pain
    symptoms. MRI should be carefully evaluated
    to rule out any radicular component.
    Treatments include muscle relaxants and
    physical therapy to break muscle spasm. If
    pain persists or if the patient is unable to continue
    with physical therapy, piriformis muscle
    injection may aid in treatment.
48
Q
  1. A 25-year-old construction worker, 8 months
    after a fall from a ladder, is unable to walk without
    assistance. However, worker compensations
    lawyers have provided video evidence of the
    patient being able to walk and also able to run
    with his dog. Which of the following is the most
    likely diagnosis?
    (A) Hypochondriasis
    (B) Factitious disorder
    (C) Malingering
    (D) Conversion disorder
    (E) Somatization disorder
A
  1. (C) There can be physical and psychologic
    symptoms of malingering and factitious disorder.
    In these conditions the patient willfully
    produces or feigns symptoms of illness or
    injury. In the factitious disorder the goal of the
    behavior is the patient’s need to be in sick
    role—a need not understood by the patient.
    Placing blood into urine and pretending to
    have posttraumatic stress disorder are examples.
    There is no apparent external goal such as
    to obtain money or drugs. It is always a psychiatric
    illness. This contrast with malingering,
    in which there is a clearly defined external goal.
    Malingering is not a psychiatric illness.
    Diagnosis of hypochondriasis require atleast
    6 month of preoccupation with the fear or
    belief that one has a serious disease, based on
    the interpretation of physical signs or sensations
    as evidence of illness. Somatization disorder
    is characterized by an extensive history
    of multiple somatic symptoms that are psychologic
    in nature. In addition to many physical
    complaints or a belief that one is sickly, the
    criteria require at least 13 symptoms from a list
    of 41. The symptom list includes 6 GI symptoms,
    7 pain symptoms, 4 cardiopulmonary
    symptoms, 12 conversion/pseudoneurologic
    symptoms, 4 sexual symptoms, and 4 female
    reproductive symptoms. Conversion disorders
    are patients presenting with physical symptoms without any anatomic or pathophysiologic
    basis (pseudoneurologic symptoms;
    pseudoparalysis, pseudoseizure etc).
49
Q
  1. A 43-year-old gentleman has developed left
    groin pain 6 months after an inguinal hernia
    repair. The patient reports pain to be severe
    stabbing pain in the left groin radiating down
    to the left testicle. On examination, the patient
    has a well-healed incision and marked cutaneous
    allodynia and hyperalgesia. This patient
    most likely has
    (A) ilioinguinal neuralgia
    (B) mesh infection
    (C) recurrent hernia
    (D) wound dehiscence
    (E) incarceration
A
  1. (A) Ilioinguinal neuralgia may develop after
    any surgery in inguinal area resulting in
    damage to the ilioinguinal nerve. Pain may
    start immediately after the surgery or may start
    after a reasonable period of healing has passed.
    Wound infection, recurrent hernia, and mesh
    infection should be ruled out to avoid any correctable
    causes of ilioinguinal pain. Pain is usually
    described as sharp, electrical sensation or
    sometimes as constant burning sensations in
    the groin area with occasional radiation into
    the testicles. Pain is exacerbated by light touch
    or rubbing of clothes. Treatment includes anticonvulsants
    and other adjuvant medications. If
    pain persists, local anesthetic diagnostic and
    therapeutic block as well as other treatment
    approaches may be warranted. Radiofrequency
    ablation, peripheral nerve stimulation, neurectomy,
    and repeat surgery should be reserved
    for resistant cases.
50
Q
  1. Which of the following is the most common
    complication from the celiac plexus block?
    (A) Hypotension
    (B) Seizure
    (C) Diarrhea
    (D) Hematoma
    (E) Subarachnoid injection
A
  1. (A)
    A. Hypotension from sympathetic blockade
    is the most common complication. It is
    important to optimally prehydrate these
    patients prior to the onset of the block.
    B. Seizure results from intravascular injection
    of large volume of local anesthetic stressing
    the need to confirm negative aspiration
    prior to injecting the solution.
    C. Diarrhea ensues as a result of sympathetic
    blockade and unopposed parasympathetic
    tone.
    D. Retroperitoneal hematoma is a rare complication
    of celiac plexus block.
    E. Subarachnoid injection is the most serious
    and very rare complication celiac block.
51
Q
  1. A patient with history of three lumbar spinal
    fusions from an injury while working in a
    halfway home who is responsive to MS Contin
    (sustained-release morphine) 30 mg, three
    times a day and Norco (hydrocodone 5 mg
    with acetaminophen 325 mg) eight tablets per
    day with adequate analgesia and improved
    functionality, but limited activity secondary to
    side effects, receives an intrathecal opioid
    pump trial after been cleared by his psychologist.
    After confirmation of appropriate placement
    of the catheter under fluoroscopy, he is
    put on 0.5 mg/d of intrathecal morphine and
    gradually escalated up to 10 mg/d because of
    inadequate analgesia. Twelve hours after the
    procedure, he complains of nausea, headache,
    and sensation of “skin peeling off his body.”
    Which of the following is the best course of
    action in this case?
    (A) Increase the intrathecal morphine until
    pain relief and resolution of symptoms
    (B) CT scan of his spine to confirm correct
    placement of the catheter
    (C) Removal of the catheter and institution
    of oral opioids
    (D) Urine toxicology
    (E) Consultation with a spine surgeon
A
  1. (D)
    A. Increase in the intrathecal morphine dose
    is warranted in some situations when a
    patient demonstrates signs and symptoms
    consistent with withdrawal or has inadequate
    analgesia. In that case it is important
    to carefully evaluate the equianalgesic
    dose accounting for change in route or
    incomplete cross tolerance with change of
    drugs. In this case, considering the oral to
    intrathecal conversion is 300 to 1, the patient
    has been escalated to 10 mg of intrathecal
    morphine a day; it seems unlikely that his
    symptoms would be because of opioid
    withdrawal provided his catheter is in the
    correct position as had been confirmed
    under fluoroscopy in this case.
    B. CT scan can be done to confirm the correct
    placement of the catheter if necessary; however
    it is highly unlikely that the catheter
    would move in a short time in a sedentary
    postsurgical patient.
    C. Removal of catheter followed by reimplantation
    is a possibility if indeed catheter is
    determined to be malpositioned. It seems
    rather premature to pursue such option at
    this time.
    D. Urine toxicology seems like a more viable
    option considering this patient’s association
    with a half way home and the time of
    onset of his symptoms approximately
    12 hours after the hospitalization. Also, the
    symptoms experienced although nonspecific,
    point toward possible withdrawal
    from a substance of abuse. It is reasonable
    to order a urine/serum toxicology screen as
    an initial step at this point while instituting
    conservative treatment with nonopioid
    analgesics and antinausea preparations.
    E. Spine surgeon consultation does not seem
    necessary at this point since the symptoms
    experienced are not truly suggestive of spinal
    hematoma, infection, or neurologic deficits
    warranting acute surgical intervention.
52
Q
303. Migraine headaches are directly related to
(A) estrogen increase
(B) estrogen decrease
(C) progesterone increase
(D) progesterone decrease
(E) none of the above
A
  1. (B) The mechanism by which ovarian hormones
    influence migraines remain to be determined,
    but an abrupt decrease in serum
    estrogen concentrations before the onset of an
    attack appears to be a critical factor. Sometimes
    the use of percutaneous estrogen gel applied
    just before and through the menstrual cycle
    may reduce the frequency of headaches.
    However, in some other cases use of low-dose
    estrogen oral contraceptive formulation are associated with a haphazard occurrence of
    attacks during the cycle, probably because of
    fluctuating estrogen levels. Therefore, it seems
    prudent to have the treatment strategies aimed
    toward preventing either a decrease or substantial
    fluctuation in the levels of estrogen.
53
Q
  1. A50-year-old female comes in complaining of
    sudden onset pain in bilateral lower extremities
    and loss of bladder function. Her physical
    examination reveals motor weakness in her left
    lower extremity 3/5 compared to the right
    along with diminished sensation to light touch,
    pinprick, and temperature along L5 and S1 dermatomes
    on the right compared to the left. Rest
    of her physical, musculoskeletal, and neurologic
    examination is normal. Lumbosacral x-rays
    done by her primary care physician demonstrate
    anterolisthesis of L5 on S1. Which of the
    following is the most appropriate immediate
    action?
    (A) Consult the spine surgeon
    (B) Intravenous Opioids
    (C) Physical therapy
    (D) Reassurance and return to home with a
    follow-up visit in 2 weeks if symptoms
    persist
    (E) A course of oral steroids
A
  1. (A)
    A. Considering the acute onset of bladder
    dysfunction and neurologic deficits on
    physical examination along with the
    anterolisthesis of L5 on S1, urgent evaluation
    by a spine surgeon seems to be the
    best immediate option of all. This patient
    needs further workup and possibly even
    urgent intervention by the spine surgeon
    at this time.
    B. While intravenous opioids can be used to
    control acute pain, they by no means
    should be considered adequate in managing
    this situation that demands urgent surgical
    attention.
    C. Physical therapy may be considered in
    future for this patient for physical rehabilitation
    once surgical evaluation and/or
    intervention has been completed. Physical
    therapy for acute pain management is inappropriate
    for this case considering the risk
    of neurologic deficits that may ensue from
    further movement of an unstable spine.
    D. This condition could be a surgical emergency
    and so this patient should be actively
    managed in an in-patient setting.
    E. Oral steroids may sometimes be beneficial
    in such setting to decrease the pain and
    inflammation associated with acute spine
    pain, but the surgical evaluation should
    take precedence over all conservative treatment
    options that may delay resolution of
    the spinal pathology.
54
Q
305. The approaches to celiac plexus block are all
EXCEPT
(A) retrocrural
(B) transcrural
(C) transaortic
(D) intercrural
(E) latera
A
  1. (E) Celiac plexus or ganglia, these terms often
    used interchangeably, are a dense network of
    pre- and postganglionic fibers. The three
    splanchnic nerves; greater, lesser, and least
    synapse at the celiac ganglia.
    A. Retrocrural approach is the most commonly
    utilized by anesthesiologists and
    considered the most traditional. The landmarks
    include iliac crests, 12th rib, dorsal
    midline, vertebral bodies (T12-L2), and lateral
    border of the paraspinal (sacrospinalis)
    muscles.
    B. Transcrural approach involves placement
    of needle tips anterior and caudal to the
    diaphragmatic crura. Advocates of this
    approach believe that this approach maximizes
    spread of injected solutions anterior
    to the aorta where the celiac plexus is most
    concentrated and this minimizes the somatic
    nerve block.
    C. Transaortic approach to celiac plexus has
    been described under both fluoroscopic
    and CT guidance. It is considered safe by
    many because of the use of single fine needle
    compared to two-needle posterior
    approach. This approach has three distinct
    advantages over the classic two-needle
    approach. First, it avoids the risk of neurologic
    complications related to posterior
    retrocrural spread of drugs. Secondly, the
    aorta provides a definitive landmark for
    needle placement when radiographic
    guidance is not available and thirdly,
    much smaller volumes of local anesthetic
    and neurolytic solutions are required to
    achieve efficacy equal to or greater than
    that of classic retrocrural approach.
    D. Intercrural approach is a term that can technically
    be applied to transaortic approach
    since the needle tips are placed in front of the
    diaphragmatic crura in this approach, but
    more commonly this term is used to refer to
    the classic anterior approach to celiac plexus
    under CT or ultrasound guidance.
    E. Lateral approach has not been described in
    literature.
55
Q
  1. A 25-year-old male presents to you with leftsided
    neck pain with radiation along lateral
    aspect of the left arm, forearm, and thumb,
    index, and middle finger. He has associated
    tingling and numbness. On neurologic examination,
    the sensation to pinprick is diminished
    in the above mentioned distribution and brachioradialis
    jerk is lost on the left compared to
    intact 2+ on the right. The MRI of C-spine is
    compatible with an acute cervical disc herniation.
    Which of the following is the most appropriate
    initial treatment?
    (A) A course of oral opioids, oral steroids,
    and spine surgical consultation
    (B) A series of cervical epidural steroid
    injections under fluoroscopy
    (C) Physical therapy
    (D) Spinal cord stimulation (SCS)
    (E) Referral to pain psychologist for coping
    strategies
A
  1. (A)
    A. The trial of oral opioids, steroids, and
    urgent consultation with a spine surgeon
    are the most appropriate initial steps in
    management of what seems to be a case of
    acute radiculopathy secondary to acute
    disc herniation. Because these substantial
    neurologic deficits may be reversed with
    appropriate and timely decompression the surgical evaluation and course of
    steroids are top priorities here.
    B. Cervical epidural steroid injections can be
    considered to decrease the inflammation,
    but does not qualify to be “most appropriate
    initial treatment.”
    C. Physical therapy can be instituted further
    down the road for rehabilitation.
    D. SCS may be beneficial to decrease neuropathic
    pain of chronic nature, but it has no
    role in an acute setting of this nature.
    E. Pain psychologist can prove to be very
    useful in patients suffering from chronic
    pain, but again has little role in acute pain
    management in this setting.
56
Q
307. Hoffmann sign is indicative of
(A) upper motor neuron lesion (UMNL)
(B) lower motor neuron lesion (LMNL)
(C) radiculopathy
(D) instability of cervical spine
(E) malingering
A
  1. (A) Hoffmann sign is indicative of UMNL. In
    fact, it is the upper extremity equivalent of
    Babinski reflex. The examiner holds the patient’s
    middle finger and briskly flicks the distal phalanx.
    A positive sign is noted if the interphalageal
    joint of thumb of the same hand flexes.
57
Q
  1. A 65-year-old male comes in complaining of
    pain in between the third and the fourth toes.
    The pain can be reproduced by palpation of
    the pulp between metatarsal heads. There is
    some relief of pain following localized administration
    of local anesthetic. Which of the following
    is the most likely diagnosis?
    (A) Plantar fascitis
    (B) Metatarsalgia
    (C) Tarsal tunnel syndrome
    (D) Morton neuroma
    (E) Painful calcaneal spur
A
  1. (D)
    A. Plantar fascitis is an inflammation of the
    tendons and the fascia of the foot as they
    insert into the calcaneal periosteum. It is
    typically seen in the people who stand on
    hardwood floors for a prolonged period of
    time. Pain is elicited with plantar compression
    over the anterior calcaneus and also
    may radiate along plantar fascia.
    B. Metatarsalgia is characterized by pain in
    the plantar surface of the metatarsal heads
    caused by prolonged weight-bearing. It
    can also be replicated with manual compression
    over the metatarsal heads. Pain is
    most commonly increased in combined
    pronation and eversion.
    C. The etiology and diagnosis of tarsal tunnel
    syndrome is somewhat controversial. This
    syndrome involves compression or inflammation
    of the posterior tibial nerve that
    provides sensory innervation to medial
    aspect of the calcaneus, motor supply to
    small lateral musculature of the foot and to
    the medial and lateral plantar branches.
    The symptoms are usually vague with
    activity related problems. Pain along with
    paresthesia, cramping, and burning is seen
    in the distribution. Palpation reveals sensitivity
    in the area. EMG testing can be utilized
    in diagnosis of tarsal tunnel syndrome
    but is controversial.
    D. Morton neuroma (interdigital neuroma) is
    the compression of the interdigital nerves
    in between the metatarsal heads and deep
    transverse metatarsal ligaments. The third
    interspace between third and fourth
    metatarsal is most frequently involved, it
    is believed to be so because lateral plantar
    nerve sends a branch to the medial plantar
    nerve to form a larger third common digital
    nerve making it less mobile. The condition
    is usually unilateral and affects
    females more commonly than men, usually
    in their 50s. The most common symptom
    is plantar pain that is increased by
    walking or by palpation between the third
    and fourth metatarsal heads.
    E. Painful calcaneal spur is often seen in morbidly
    obese people or those who stand or
    walk excessively. Pain is increased in the
    morning or after a prolonged rest and similar
    to plantar fascitis except that it is more
    predominant in the posterior aspect of the
    plantar calcaneus.
58
Q
309. Which of the following is the most common
nerve missed with the interscalene brachial
plexus nerve block?
(A) Ulnar
(B) Radial
(C) Musculocutaneous
(D) Median
(E) Axillary
A
  1. (A) Interscalene block of brachial plexus is
    especially effective for surgery of the shoulder
    or upper arm, as the roots of the brachial plexus
    are most easily blocked with this technique.
    Ulnar nerve is most frequently spared since it
    is derived from the eighth cervical nerve and
    the block is placed at a more cephalic site with
    this approach. This block is ideal for reduction
    of a dislocated shoulder or any other type of
    surgery on shoulder or upper arm.
59
Q
  1. A 23-year-old gymnast while performing a
    double loop hears a popping sound in her left
    knee. Her knee immediately swells up and is
    very painful. On physical examination, tenderness
    on palpation and effusion is demonstrated.
    McMurray test is positive. Which of
    the following is the most likely diagnosis?
    (A) Baker cyst
    (B) Anterior cruciate ligament tear
    (C) Posterior cruciate ligament tear
    (D) Torn medial meniscus
    (E) Pes anserine bursitis
A
  1. (D)
    A. A baker (popliteal) cyst represents ballooning
    of the synovium-lined joint capsule,
    usually on the posteromedial aspect
    of the knee. It is usually a secondary manifestation
    of underlying condition that
    causes chronic inflammation of the knee,
    such as meniscal tear, knee synovitis or
    intra-articular loose body. The diagnosis of
    the popliteal cyst can be made by direct
    palpation of the mass. Arthrography or an
    MRI can verify the diagnosis and demonstrate
    its communication with the joint
    cavity. The cyst usually resolves with correction
    of the underlying pathology.
    B. Anterior cruciate ligament is the most
    commonly injured knee ligament in athletes.
    Injury to this ligament will result in a
    bloody knee effusion that is very indicative
    of this particular kind of injury. Three
    tests used to diagnose anterior cruciate ligament
    injury are anterior drawer test,
    Lachman test, and pivot shift test.
    C. Posterior cruciate ligament is usually damaged
    in violent, usually high–kinetic
    energy injuries. These usually occur in
    combination with fractures, specifically to
    the patella and hip or with other knee ligament
    injuries. Injury to popliteal artery
    should be evaluated in this injury with
    palpation or even arteriography. The test
    used to diagnose posterior cruciate ligament
    injury is posterior drawer test.
    D. In stance more than 60% of the body’s
    weight is carried on the peripheral aspect
    of the tibial plateau by meniscal fibrocartilages.
    In younger persons, the meniscal
    injuries usually accompany other ligament
    injuries whereas in elderly, these usually
    occur in isolation. When a meniscal tear is
    extensive it can result in block to terminal
    knee flexion or extension, commonly
    described by patients as “locking of the
    knee”. A torn meniscus can cause knee
    swelling and pain as it irritates the joint
    surface or synovium. Chronic meniscal
    injuries can result in arthritic joint surface.
    Joint line tenderness is found in about 50%
    of these injuries. McMurray test is used to
    detect tear of the meniscus that can be displaced.
    It is performed by flexing and
    extending the knee between 90° and 140°
    of flexion. One of the examiner’s hands
    rotates the tibia at the ankle while the
    other hand is placed in front of the joint
    line. This is followed by the extension of
    knee in the rotated position. A palpable
    click indicates an unstable tear of the
    meniscus. The Apley grind test can help
    distinguish between tear in the anterior or
    posterior portion of the meniscus. MRI or
    arthroscopy can also be used as diagnostic
    tools to identify a meniscal lesion.
    E. Pes anserine bursa lies between the medial
    hamstring tendons (sartorius, gracilis, and
    semitendinosus) and proximal medial
    tibia. It is inferior to the joint line which
    helps distinguish from the medial joint
    line tenderness secondary to meniscal
    injury.
60
Q
  1. A 35-year-old female is rear ended at 45 mph
    resulting in acute neck pain that was diagnosed
    to be of musculoskeletal nature in the emergency
    room. On the next day, her symptoms
    progress to right upper extremity pain and
    weakness, both of which are exacerbated with
    ipsilateral flexion of her neck and reaching
    overhead. She has no neurologic deficits and
    MRI of her neck shows no obvious pathology.
    There is obliteration of the radial pulse with
    arm extension and abduction. Which of the following
    is the most likely diagnosis?
    (A) Brachial plexitis
    (B) Cervical degenerative disc disease
    (C) Whiplash injury
    (D) Pancoast tumor
    (E) Thoracic outlet syndrome
A
  1. (E)
    A. Brachial plexitis is an acute disorder of that
    almost always begins with unilateral diffuse
    pain in the shoulder followed by
    weakness in the proximal muscles. Sensory
    disturbances are less pronounced than motor
    deficits. The pain usually subsides after the
    acute phase. Electrodiagnostic studies can
    help to establish the diagnosis.
    B. Cervical degenerative disc disease can result
    in diffuse axial pain in the neck or radicular
    pain along a particular dermatome corresponding
    to the nerve root involved if associated
    with a herniated nucleus pulposus.
    C. Whiplash injury typically follows a highimpact
    motor vehicle accident that results
    in axial neck pain. It has a musculoskeletal
    component to it and is frequently associated
    with facet joint involvement.
    D. Pancoast tumor is the tumor of the apex of
    the lung that typically involves the
    brachial plexus. Pain is a common presenting
    symptom usually involving the lower
    cervical nerve roots or trunks. CT scan or
    MRI can sometimes offer valuable diagnostic
    information.
    E. Thoracic outlet syndrome usually involves
    impingement of subclavian vessels and
    lower trunk of brachial plexus resulting in
    various degrees of vascular or neurologic
    compromise or both with local supraclavicular
    pain. The most common etiologies are
    cervical rib, hypertrophy of scalenus anticus,
    costoclavicular abnormalities, but nevertheless
    can result from an acute trauma.
    The pain and sensory changes are usually
    aggravated by any activity that extends the
    brachial plexus, including carrying heavy
    objects, abducting arms over the head or
    with repetitive movements of the arm.
    Motor weakness is seen in intrinsic muscles
    of the hand. The obliteration of radial
    pulse with arm extension or abduction or
    traction can be present and is called Adson
    or Allen test.
61
Q
  1. The following is true about the H reflex EXCEPT
    (A) in clinical practice H reflex is limited to
    calf muscles
    (B) it is recorded in gastrocnemius and
    soleus muscles by stimulating the posterior
    tibial nerve in the popliteal fossa
    (C) because of the distance the impulse
    travels, the latency of the H wave is
    shorter than the F wave
    (D) the H reflex recorded from the soleus
    muscle is primarily mediated by the
    S1 nerve root
    (E) H reflex is normal in L5 radiculopathy
    whereas is prolonged in S1 radiculopathy
A
  1. (C)
    A. H wave responses, in adults can be obtained
    in lower extremities. H wave response is an
    electric equivalent of the ankle deep tendon
    reflex, when the tibial nerve is stimulated.
    B. The tibial nerve behind the knee in the
    popliteal fossa is stimulated and the impulse
    travels via afferent fibers to the spinal cord
    at the S1 level. After synapse in the cord,
    anterior horn cells produce a motor response
    that can be recorded in gastrocnemius and
    soleus muscles.
    C. H waves are true reflexes, F wave is not.
    Because the H wave has to travel to the
    level of cord in order to produce a response,
    the latency is longer compared to F wave.
    D. This is correct as explained in (B).
    E. Since the impulses are conducted through
    S1 nerve, H reflex is typically prolonged in
    S1 radiculopathy but may be normal in L5
    radiculopathy.
62
Q
313. The arteria radicularis magna, also known as
artery of Adamkiewicz arises from aorta, at the
following spinal levels:
(A) L4-5
(B) T9-12
(C) T5-8
(D) T11-12
(E) T5-9
A
  1. (B) The spinal cord receives its blood supply
    from three longitudinal arteries: a single anterior
    spinal artery and two posterior spinal arteries.
    The diameter of anterior spinal artery is
    greatest at the cervical and lower thoracic levels
    and narrowest at the midthoracic levels from
    T3-T9. This region of the cord is considered
    to be the “vulnerable zone” with respect to
    circulation. The anterior spinal artery is reinforced
    at a number of segmental levels by
    feeder arterial branches called anterior
    medullary feeder arteries. At the thoracic level,
    there are a total of eight of these feeder arteries,
    largest of which is called artery of Adamkiewicz
    or great anterior medullary artery. This artery
    typically enters the cord on the left side anywhere
    from T7 to L4, but most commonly at
    T9-T12.
63
Q
  1. A56-year-old male who is an avid golfer comes
    in with left elbow pain not relieved after antiinflammatory
    medication trial, warm compress,
    and physical therapy. He has not been
    able to play 18 holes recently and this is making
    him quite depressed. On examination, passive
    flexion or extension against resistance of his
    left wrist causes pain. Which of the following is
    the most probable diagnosis in this patient?
    (A) Posterior interosseous nerve entrapment
    (B) Medial epicondylitis
    (C) Lateral epicondylitis
    (D) de Quervain disease
    (E) Brachioradialis tendonitis
A
  1. (C)
    A. The involvement of deep radial nerve is
    called posterior interosseous nerve entrapment.
    The symptoms are similar to radial
    tunnel syndrome including pain over the
    proximal dorsal forearm, with maximum
    tenderness at the site of radial tunnel that
    is 4 cm distal to the lateral epicondyle over
    the posterior interosseous nerve. The pain
    is typically elicited by attempting to resist
    extension of long finger.
    B. Medial epicondylitis or golfer’s elbow
    results in pain and exquisite tenderness over
    medial epicondyle that is further aggravated
    by flexion and pronation of the forearm
    and the wrist.
    C. Lateral epicondylitis or tennis elbow
    involves the extensor-supinator muscle
    mass, including extensor carpi radialis brevis,
    extensor digitorum communis, extensor
    carpi radialis longus, extensor carpi ulnaris,
    and supinator. The extensor carpi radialis is
    most commonly involved, mostly from
    repetitive movement of the wrist involving
    wrist flexion, elbow extension, and forearm
    pronation. Provocative test involves grasping
    or extending the wrist against resistance
    or supinating the forearm when sudden and
    severe pain is experienced in the area of lateral
    epicondyle. The patient’s being an
    “avid golfer” is a distractor here.
    D. de Quervain disease or tenosynovitis of the
    tendon sheath of extensor pollicis brevis and
    adductor pollicis longus causes swelling and
    tenderness of anatomic snuff box.
    E. Brachioradialis tendonitis results in pain
    in the lateral forearm, that is, region of brachioradialis
    tendon, the provocative tests
    described above typically do not elicit
    characteristic symptoms.
64
Q
  1. All of these cervical pathologies are seen in
    patients with rheumatoid arthritis EXCEPT
    (A) subaxial subluxation
    (B) cranial settling
    (C) posterior-longitudinal ligament
    thickening
    (D) atlantoaxial subluxation
    (E) instability of cervical-zygapophyseal
    joints
A
  1. (C) Patients with cervical rheumatoid arthritis
    develop neck pain exacerbated by movement,
    with atlantoaxial disease producing pain in
    upper cervical spine and subaxial involvement
    producing pain in lower neck and clavicular
    areas. Neurologic involvement is seen in more
    advanced cases of spinal cord or nerve root
    compromise related to deformity and soft tissue hypertrophy. Plain radiography is useful
    in showing structural abnormalities and
    dynamic studies including flexion extension,
    oblique and open mouth frontal projections in
    identifying instability. Anterior subluxation of
    atlantoaxial joint is the most common form of
    cervical spine derangement followed by subaxial
    subluxation (between C3 and C7), lateral
    subluxation, cranial settling (vertical subluxation),
    and posterior subluxation. Also, the
    autoimmune inflammatory changes affect the
    synovium of zygapophyseal joints resulting in
    laxity and subsequent instability.
65
Q
  1. While undergoing lumbar sympathetic block
    for CRPS, patient complains of sudden onset of
    sharp ipsilateral groin and genital pain on injection
    of the contrast agent. Which of the following
    is the most likely cause of this symptom?
    (A) Trauma to L2 nerve root
    (B) Trauma to genitofemoral nerve
    (C) Psoas spasm
    (D) Epidural injection
    (E) Successful lumbar sympathetic block
A
  1. (B)
    A. Trauma to L2 nerve root may cause ipsilateral
    groin pain, but is not the most likely cause.
    B. The most likely cause of the symptoms
    mentioned in the question is trauma to genitofemoral
    nerve. In fact, it is the most common
    complication associated with lumbar
    sympatholysis, particularly by the lateral
    approach. The incidence has been reported to
    be as high as 15%, but may be as low as 4%
    with a single-needle technique. Most cases
    are transient and resolve with conservative
    measures but others may last as long as 6
    weeks. Repeat local anesthetic lumbar sympathetic
    block, TENS (transcutaneous electrical
    nerve stimulator) unit and intravenous
    lidocaine have all been described as options
    for remission of genitofemoral neuralgia.
    C. Psoas spasm is also sometimes seen but it
    typically produces discomfort in ipsilateral
    low back.
    (D) and (E) do not present as groin pain.
66
Q
317. Which of the following is the most common
inherited neuropathy?
(A) Familial amyloid polyneuropathy
(B) Fabry disease
(C) Porphyric neuropathy
(D) Charcot-Marie-Tooth disease
(E) Diabetic polyneuropathy
A
  1. (D) Painful symptoms of Charcot-Marie-Tooth
    (CMT) disease have been described in the
    hypertrophic or demyelinating form (CMT-1).
    Pain may be described shooting, sharp, or
    burning in their toes, feet, ankles, and knees.
    Common presentation is in the first or second
    decade with difficulties walking or running.
67
Q
318. A 52-year-old man comes to your office complaining
of 11/2 years of “burning” pain in the
metatarsal areas of his left foot. Which of the
following is the most likely diagnosis?
(A) Posterior tibial neuritis
(B) Plantar fasciitis
(C) Morton neuroma
(D) Tarsal tunnel syndrome
(E) Hallux rigidus
A
  1. (C) Morton neuroma may be considered in the
    spectrum of interdigital neuritis (compression
    neuropathy). It is usually between the third
    and fourth toes or less often between the fourth and fifth toes. The pain tends to be experienced
    more with walking and weight bearing while
    wearing shoes. The pain is generally alleviated
    with rest and removal of shoes. The pain may
    be reproduced by exerting pressure between
    the two toes implicated. Interdigital injection of
    local anesthetic relieves the pain.
68
Q
  1. In MRI of the lumbar spine T2-weighted
    images
    (1) are generally more time-consuming to
    obtain
    (2) are ideal to image the anatomic detail of
    end-plate reactive changes
    (3) exhibit increased sensitivity to higher
    water content and thus, may be useful
    in imaging infectious processes or
    inflammation
    (4) can be used in place of gadolinium-
    DTPA (diethylenetriamine pentaacetic
    acid) contrast in imaging of postoperative
    patients to differentiate scarring
    from intervertebral disc issues
A
  1. (B) MRI, especially with T2-weighted images
    (though generally more time consuming to
    obtain) is useful in imaging conditions such as
    osteomyelitis, discitis, spinal cord compression,
    and malignancy. T1-weighted images provide
    reasonably good anatomic detail in imaging of
    end-plate reactive changes as well as postoperative
    scarring, but gadolinium-DTPA contrast
    should be used in postoperative patients
    to differentiate scarring from intervertebral
    discs.
69
Q
  1. In EMG and NCS, the H reflex
    (1) is the electrical equivalent of a muscle
    stretch reflex elicited by tendon tap
    (2) is mostly present in the soleus muscle
    but at times also can be elicited in the
    forearm flexor muscles
    (3) may be delayed or absent in S1 radiculopathy
    (4) latencies are length-dependent and
    should be adjusted for patient’s height
A
  1. (E) The H reflex is examined utilizing a modified
    motor nerve conduction study technique.
    The H reflex is generally present in the soleus
    muscle and at time forearm flexor muscles. It
    may be more widespread in hyperreflexic conditions
    (eg, myelopathy) and pediatrics.
    Delayed or absence of the tibial H wave may
    reflect S1 radiculopathy or other neuropathic
    processes.
70
Q
  1. A previously healthy 83-year-old male presents
    to your office complaining of acute
    abdominal pain but without obvious etiology.
    Medical conditions which should be investigated
    include
    (1) pneumonia
    (2) inflammatory bowel disease
    (3) pyelonephritis
    (4) inferior wall myocardial infarction
A
  1. (E) The elderly may seek medical attention for
    multiple problems with initial complaints of
    abdominal pain including: dissecting abdominal
    aortic aneurysm in diabetic ketoacidosis,
    pneumonia, pyelonephritis, inflammatory
    bowel disease, mesenteric ischemia, constipation,
    bowel obstruction, peritonitis, and druginduced
    GI mucosal irritation.
71
Q
322. Patients diagnosed with cubital tunnel syndrome
may have
(1) pain and numbness in the ulnar border
of the forearm and hand
(2) clawing of the small finge
(3) Wartenberg sign
(4) a deep aching sensation in the mid
forearmr
A
  1. (A) The ulnar nerve may be compressed in the
    cubital tunnel (cubital tunnel syndrome) which
    may lead to atrophy of the first dorsal
    interosseous muscle, clawing of the small
    finger, weakness of small finger adduction
    (Wartenberg sign) and eventually in chronic
    ulnar nerve compromise—with weakness of
    grip and pinch.
72
Q
  1. A 53-year-old male comes to your office complaining
    of foot pain (predominantly in the
    heel—but also with diffuse plantar symptoms)
    which also occurs at night and can be exacerbated
    by prolonged standing or walking. It is
    associated with weakness of the phalanges
    (impairing the pushing off phase of walking) as
    well as sensory loss and paresthesia. After a
    complete history and physical examination are
    completed, the differential diagnosis may
    include
    (1) plantar fasciitis
    (2) peripheral neuropathies
    (3) posterior tibial nerve entrapment
    (4) tarsal tunnel syndrome
A
  1. (E) Tarsal tunnel syndrome is not a common
    source of foot discomfort and needs to be distinguished from multiple other causes of pain
    in the foot including: painful peripheral neuropathies,
    medial plantar nerve entrapment (which
    may occur in joggers), posterior tibial nerve entrapment
    symptoms tend to be located in medial plantar
    heel area, abductor digiti quinti nerve
    entrapment (usually with burning pain in heel pad
    area), and plantar fasciitis. Plantar fasciitis pain
    may be diffuse or migrate but with time is usually
    noted at the inferior aspect of the heel (around the
    medial calcaneal tuberosity) mainly, although typically
    severe with the first few steps in the morning,
    tends to diminish through the course of the day
    (unless intense or prolonged weight-bearing activity
    is under taken).
73
Q
  1. The diagnostic criteria for CRPS I—as accepted
    in 1994 by the International Association for the
    Study of Pain (IASP)—includes which of the
    following?
    (1) The presence of an initiating noxious
    event, or a cause of immobilization
    (2) Continuing pain, allodynia, or hyperalgesia
    with which the pain is disproportionate
    to any inciting event
    (3) Evidence at some time of edema,
    changes in skin blood flow, or abnormal
    sudomotor activity in the region of pain
    (4) This diagnosis is excluded by the existence
    of conditions that would otherwise
    account for the degree of pain and
    dysfunction
A
  1. (E) Although, somewhat controversial and different
    from various proposed research criteria,
    the diagnosis of CRPS I, includes:
  2. The presence of an initiating noxious event
    or a cause of immobilization.
  3. Continuing pain, allodynia, or hyperalgesia
    with which the pain is disproportionate
    to any inciting event.
  4. Evidence at some time of edema, changes
    in skin blood flow, or abnormal sudomotor
    activity in the region of the pain.
  5. This diagnosis is excluded by the existence
    of condition that otherwise would account
    for the degree of pain and dysfunction.
74
Q
  1. The paroxysmal hemicranias are rare benign
    headache disorders that may typically be associated
    with
    (1) conjunctival injection
    (2) rhinorrhea
    (3) ptosis
    (4) eyelid edema
A
  1. (E) Paroxysmal hemicranias may be chronic
    (CPH) (eg, daily) or episodic (EPH) (eg, discrete
    headache period or separated by periods
    of remission) characterized by severe, excruciating,
    throbbing, boring, or pulsatile pain
    affecting the orbital, supraorbital, and temporal
    regions.
    The pain tends to be associated with at
    least one of the following signs or symptoms
    ipsilateral to the painful side:
  2. Conjunctival injection
  3. Nasal congestion
  4. Lacrimation
  5. Ptosis
  6. Rhinorrhea
  7. Eyelid edema
    Attacks may occur at any time—occasionally
    waking patients from sound sleep and
    tend to last for 2 to 25 minutes (although may
    linger a couple of hours). The patient generally
    has 1 to 40 attacks per day.
75
Q
326. Which of the following statement(s) is (are)
true?
(1) The most common cause of thoracic
radiculopathy is diabetes mellitus
(2) The most common levels affected by
herniated disc at cervical level are C4-5,
C5-6, and C6-7
(3) L4-5 disc is more commonly herniates
than L5-S1
(4) The nerve roots involved most commonly
in thoracic outlet syndromes are
C8 and T1
A
  1. (E)
  2. Although thoracic radiculopathy has been
    described to result from multiple etiologies
    including tumor, scoliosis, infection, spondylosis,
    and herniated disc, diabetes mellitus is
    described as the most common cause.
  3. The lower cervical discs are most commonly
    affected by herniation.
  4. The frequency of L4-5 herniation is 45%
    compared to 42% at the level of L5-S1. With
    L4-5 herniation, L5 nerve root is most commonly
    affected.
  5. Lower cervical nerve roots of brachial
    plexus, that is, C8 and T1 nerve roots are
    most commonly affected in thoracic outlet
    syndrome.
76
Q
327. The characteristics of conus medullaris syndrome
include
(1) asymmetric paraplegia
(2) symmetric paraplegia
(3) bladder function preservation
(4) upper motor neuron lesion signs
A
  1. (C) Epidural spinal cord compression is compression
    of spinal cord or cauda equina nerve
    roots from a lesion outside the dura mater.
    Epidural spinal cord or cauda equina compression
    is the second most common neurologic complication
    of cancer, occurring in up to 10% of
    patients. The most common tumors causing
    metastatic epidural compression are breast, lung,
    prostate, lymphoma, sarcoma, and kidney. Conus
    medullaris lesions typically cause a rapidly progressive
    symmetric perineal pain followed by
    early autonomic dysfunction, saddle sensory loss,
    and motor weakness. Limited straight leg raise
    test usually points to an epidural or intradural
    extramedullary lesion causing root compression,
    whereas segmental pain and sacral sparing suggest
    intramedullary disease.
77
Q
  1. Which of the following statement(s) is (are)
    true for central pain of spinal cord origin?
    (1) Most common etiology is of traumatic
    origin
    (2) Most common type of pain in these
    patients is spontaneous steady, burning,
    or dysesthetic pain affecting approximately
    96% of patients
    (3) Bowel and bladder dysfunction can be
    seen in these patients
    (4) Most patients will develop cord central
    pain within 1 to 6 months of causative
    lesion although some may present more
    than 5 years out
A
  1. (E)
  2. The incidence of spinal cord pain has been
    estimated to be in the range of 6.4% to 94% of
    patients who experience spinal cord injury.
  3. Patients may describe a variety of pain types;
    however, the three most common types are
    spontaneous steady, spontaneous neuralgic, and evoked pain including allodynia and
    hyperpathia. According to a study of 127
    patients with spinal cord pain by Boureau
    and colleagues, 75% of patients reported
    burning pain.
  4. Bowel and bladder dysfunction may be
    associated with spinal cord injury depending
    on the level and extent of injury.
  5. Onset is typically within 1 to 6 months of
    the injury. When the onset was delayed
    beyond 1 year, 56% of the patients were
    found to suffer from a syrinx.
78
Q
  1. A positive Froment sign indicates which of
    the following?
    (1) Weakness of first dorsal interosseous
    (2) Weakness of flexor pollicis brevis
    (3) Weakness of adductor pollicis
    (4) Weakness of hypothenar muscles
A
  1. (A) Froment sign is positive when ulnar nerve
    dysfunction is present. Froment sign is tested
    by placing a piece of paper between patient’s
    thumb and index finger and checking the position
    of the thumb as the examiner tries to pull
    the paper away from the patient. Normally the
    distal joint of the thumb remains in extension
    but if there is ulnar nerve dysfunction the tip of
    the thumb flexes significantly to increased pressure
    in attempt to keep the paper from moving.
79
Q
330. The potential for drug-induced painful neuropathies
exist with which of the following
agents?
(1) Amiodarone
(2) Metronidazole
(3) Pyridoxine
(4) Vincristine
A
  1. (E) Drug-induced painful neuropathies may
    include toxoids (especially with doses greater
    than 200 mg/m2), cisplatinum, vincristine, amiodarone,
    metronidazole, and pyridoxine (especially
    at doses greater than 200-300 mg/d).
80
Q
331. Spinal cord stimulation (SCS) has been used
for the treatment of
(1) failed back surgery syndrome
(2) CRPS
(3) angina
(4) peripheral vascular disease
A
  1. (E) SCS has been utilized by clinicians for a variety
    of chronic pain issues. Although a large body
    of work has been published, precise mechanisms
    of action of SCS remain elusive. Animal
    studies suggest that SCS triggers release of serotonin,
    substance P, and γ-aminobutyric acid
    (GABA) within the spinal cord dorsal horn.